Pediatric Success Respiratory, Pediatrics: Gastrointestinal, Pediatrics: Genitourinary, sfg

Réussis tes devoirs et examens dès maintenant avec Quizwiz!

The nurse is caring for an infant with biliary atresia. The parents ask why the child is receiving cholestyramine. Select the nurse's best response. 1. To lower your child's cholesterol. 2. To relieve your child's itching. 3. To help your child gain weight. 4. To help feedings be absorbed in a more efficient manner.

2. The primary reason cholestyramine is administered to the child with biliary atresia is to relieve pruritus.

Which should the nurse administer to provide quick relief to a child with asthma who is coughing, wheezing, and having difficulty catching her breath? 1. Prednisone. 2. Singulair (montelukast). 3. Albuterol. 4. Flovent (fluticasone).

3. Albuterol is the quick-relief bronchodilator of choice for treating an asthma attack. TEST-TAKING HINT: The test taker must know the medications used to treat asthma and which are used in which situations.

The nurse is reviewing discharge instructions with the parents of a child who had a tonsillectomy 24 hours ago. The parents tell the nurse that the child is a big eater, and they want to know what foods to give the child for the next 24 hours. What is the nurse's best response? 1. "The child's diet should not be restricted at all." 2. "The child's diet should be restricted to clear liquids." 3. "The child's diet should be restricted to ice cream and cold liquids." 4. "The child's diet should be restricted to soft foods."

4. Soft foods are recommended to limit the child's pain and to decrease the risk for bleeding. TEST-TAKING HINT: The test taker can eliminate answer 1 by knowing there are usually some dietary restrictions following any surgical procedure.

The nurse is interviewing the parents of a 6-year-old who has been experiencing constipation. Which could be a causative factor? Select all that apply. 1. Hypothyroidism. 2. Muscular dystrophy. 3. Myelomeningocele. 4. Drinks a lot of milk. 5. Active in sports.

1, 2, 3, 4. 1. Hypothyroidism can be a causative factor in constipation. 2. Weakened abdominal muscles can be seen in muscular dystrophy and can lead to constipation. 3. Myelomeningocele affects the innerva- tion of the rectum and can lead to constipation. 4. Excessive milk consumption can lead to constipation. 5. Activity tends to decrease constipation and increase regularity.

The nurse is caring for an infant newly diagnosed with Hirschsprung disease. What does the nurse understand about this infant's condition? 1. There is a lack of peristalsis in the large intestine and an accumulation of bowel contents, leading to abdominal distention. 2. There is excessive peristalsis throughout the intestine, resulting in abdominal distention. 3. There is a small-bowel obstruction leading to ribbon-like stools. 4. There is inflammation throughout the large intestine, leading to accumulation of intestinal contents and abdominal distention.

1. In Hirschsprung disease, a portion of the large intestine has an area lacking in ganglion cells. This results in a lack of peristalsis as well as an accumula- tion of bowel contents and abdominal distention.

A child receiving peritoneal dialysis has not been having adequate volume in the return. The child is currently edematous and hypertensive. Which would the nurse anticipate the physician to do? 1. Increase the glucose concentration of the dialysate. 2. Decrease the glucose concentration of the dialysate. 3. Administer antihypertensives and diuretics but not change the dialysate concentration. 4. Decrease the dwell time of the dialysate.

1. Increasing the concentration of glucose will pull more fluid into the return.

The parent of a child with influenza asks the nurse when the child is most infectious. Which is the nurse's best response? 1. "Twenty-four hours before and after the onset of symptoms." 2. "Twenty-four hours after the onset of symptoms." 3. "One week after the onset of symptoms." 4. "One week before the onset of symptoms."

1. Influenza is most contagious 24 hours before and 24 hours after onset of symptoms. TEST-TAKING HINT: This question requires the test taker to have knowledge of the communicability of influenza.

Which is the nurse's best response to a parent who asks what can be done at home to help a child with upper respiratory infection (URI) symptoms and a fever get better? 1. "Give your child small amounts of fluid every hour to prevent dehydration." 2. "Give your child Robitussin at night to reduce his cough and help him sleep." 3. "Give your child a baby aspirin every 4 to 6 hours to help reduce the fever." 4. "Give your child an over-the-counter cold medicine at night."

1. It is essential that parents ensure their children remain hydrated during a URI. The best way to accomplish this is by giving small amounts of fluid frequently. TEST-TAKING HINT: The test taker can eliminate answers 2 and 4 because overthe- counter cold and cough medications are not recommended for infants.

The nurse is caring for a newborn with esophageal atresia. When reviewing the mother's history, which would the nurse expect to find? 1. Maternal polyhydramnios. 2. Pregnancy lasting more than 38 weeks. 3. Poor nutrition during pregnancy. 4. Alcohol consumption during pregnancy.

1. Maternal polyhydramnios is present because the infant cannot swallow and absorb the amniotic fluid in utero.

The parent of a child being evaluated for celiac disease asks the nurse why it is important to make dietary changes. Select the nurse's best response. 1. "The body's response to gluten causes damage to the mucosal cells in the intestine, leading to absorption problems." 2. "The body's response to consumption of anything containing gluten is to create special cells called villi, which leads to more diarrhea." 3. "The body's response to gluten causes the intestine to become more porous and hang on to more of the fat-soluble vitamins, leading to vitamin toxicity." 4. "The body's response to gluten causes damage to the mucosal cells, leading to malabsorption of water and hard, constipated stools."

1. The inability to digest protein leads to an accumulation of an amino acid that is toxic to the mucosal cells and villi, leading to absorption problems.

The parents of a child with glomerulonephritis ask how they will know their child is improving after they go home. Which is the nurse's best response? 1. "Your child's urine output will increase, and the urine will become less tea-colored." 2. "Your child will rest more comfortably as lab tests become more normal." 3. "Your child's appetite will decrease as urine output increases." 4. "Your child's laboratory values will become more normal."

1. When glomerulonephritis is improv- ing, urine output increases, and the urine becomes less tea-colored. These are signs that can be monitored at home by the child's parents.

The nurse is providing discharge teaching to the parents of an infant with an umbilical hernia. Which should be included in the plan of care? 1. If the hernia has not resolved on its own by the age of 12 months, surgery is generally recommended. 2. If the hernia appears to be more swollen or tender, seek medical care immediately. 3. To help the hernia resolve, place a pressure dressing over the area gently. 4. If the hernia is repaired surgically, there is a strong likelihood that it will return.

2. If the hernia appears larger, swollen, or tender, the intestine may be trapped, which is a surgical emergency.

The nurse is providing discharge instructions to the parents of an infant born with bladder exstrophy who had a continent urinary reservoir placed. Which statement should be included? 1. "Allow your child to sleep on the abdomen to provide comfort during the immediate post-operative period." 2. "As your child grows, be cautious around playgrounds because the surface could be a health hazard." 3. "As your child grows, be sure to encourage many different foods because it is not likely that food allergies will develop." 4. "Encourage your child's development, by having brightly colored objects around, such as balloons."

2. Many children with urological malfor- mations are prone to latex allergies. The surfaces of playgrounds are often made of rubber, which contains latex.

The nurse in a diabetic clinic sees a 10-year-old who is a new diabetic and has had trouble maintaining blood glucose levels within normal limits. The child's parent states the child has had several daytime "accidents." The nurse knows that this is referred to as which of the following? 1. Primary enuresis. 2. Secondary enuresis. 3. Diurnal enuresis. 4. Nocturnal enuresis.

2. Secondary enuresis refers to urinary incontinence in a child who previously had bladder control.

Which best describes the electrolyte imbalance that occurs in chronic renal failure (CRF)? 1. Decreased serum phosphorus and calcium levels. 2. Depletion of phosphorus and calcium stores from the bones. 3. Change in the structure of the bones, causing calcium to remain in the bones. 4. Nutritional needs are poorly met, leading to a decrease in many electrolytes such as calcium and phosphorus.

2. The calcium and phosphorus levels are drawn from the bones in response to low calcium levels.

A 5-year-old is brought to the ER with a temperature of 99.5°F (37.5°C), a barky cough, stridor, and hoarseness. Which nursing intervention should the nurse prepare for? 1. Immediate IV placement. 2. Respiratory treatment of racemic epinephrine. 3. A tracheostomy set at the bedside. 4. Informing the child's parents about a tonsillectomy.

2. The child has stridor, indicating airway edema, which can be relieved by aerosolized racemic epinephrine. TEST-TAKING HINT: The test taker must accurately identify that the question is describing a child with croup and know the accepted treatments.

The nurse is providing discharge instructions to the parents of a child who had an appendectomy for a ruptured appendix 5 days ago. The nurse knows that further education is required when the parent states: 1. "We will wait a few days before allowing our child to return to school." 2. "We will wait 2 weeks before allowing our child to return to sports." 3. "We will call the pediatrician's office if we notice any drainage around the wound." 4. "We will encourage our child to go for walks every day."

2. The child should wait 6 weeks before returning to any strenuous activity.

The nurse is caring for a neonate with an anorectal malformation. The nurse notes that the infant has not passed any stool per rectum but the infant's urine contains meconium. The nurse can make which assumption? 1. The child likely has a low anorectal malformation. 2. The child likely has a high anorectal malformation. 3. The child will not need a colostomy. 4. This malformation will be corrected with a nonoperative rectal pull-through.

2. The presence of stool in the urine indicates that the anorectal mal - formation is high.

The parents of a 6-week-old male ask the nurse if there is a difference between an inguinal hernia and a hydrocele. Which is the nurse's best response? 1. "The terms are used interchangeably and mean the same thing." 2. "The symptoms are similar, but an inguinal hernia occurs when tissue protrudes into the groin, whereas a hydrocele is a fluid-filled mass in the scrotum." 3. "A hydrocele is the term used when an inguinal hernia occurs in females." 4. "A hydrocele presents in a manner similar to that of an inguinal hernia but causes increased concern because it is often malignant."

2. The symptoms are similar, but an inguinal hernia occurs when tissue pro- trudes into the groin, and a hydrocele is a fluid-filled mass in the scrotum.

Which needs to be present to diagnose hemolytic uremic syndrome (HUS)? 1. Increased red blood cells with a low reticulocyte count, increased platelet count, and renal failure. 2. Decreased red blood cells with a high reticulocyte count, decreased platelet count, and renal failure. 3. Increased red blood cells with a high reticulocyte count, increased platelet count, and renal failure. 4. Decreased red blood cells with a low reticulocyte count, decreased platelet count, and renal failure.

2. The triad in HUS includes decreased red blood cells (with a high reticulo- cyte count as the body attempts to produce more red blood cells), decreased platelet count, and renal failure.

The parents overhear the health-care team refer to their child's disease as in stage III. The parents ask the nurse what this means. Which is the nurse's best response? 1. The tumor is confined to the abdomen, but it has spread to the lymph nodes or peritoneal area; the prognosis is poor. 2. The tumor is confined to the abdomen, but it has spread to the lymph nodes or peritoneal area; the prognosis is very good. 3. The tumor has been found in three other organs beyond the peritoneal area; the prognosis is good. 4. The tumor has spread to other organs beyond the peritoneal area; the prognosis is poor.

2. The tumor is confined to the abdomen but has spread to the lymph nodes or peritoneal area. The prognosis is still very good.

The nurse anticipates that the child who has had a kidney removed will have a high level of pain and will require invasive and noninvasive measures for pain relief. The nurse anticipates that the child will have pain because of which of the following? 1. The kidney is removed laparoscopically, and there will be residual pain from accumulated air in the abdomen. 2. There is a post-operative shift of fluids and organs in the abdominal cavity, leading to increased discomfort. 3. The chemotherapy makes the child more sensitive to pain. 4. The radiation therapy makes the child more sensitive to pain.

2. There is a post-operative shift of fluids and organs in the abdominal cavity, leading to increased discomfort.

The nurse is administering Prilosec (omeprazole) to a 3-month-old with gastro - esophageal reflux (GER). The child's parents ask the nurse how the medication works. Select the nurse's best response. 1. "Prilosec is a proton pump inhibitor that is commonly used for reflux in infants." 2. "Prilosec decreases stomach acid, so it will not be as irritating when your child spits up." 3. "Prilosec helps food move through the stomach quicker, so there will be less chance for reflux." 4. "Prilosec relaxes the pressure of the lower esophageal sphincter."

2. This accurate description gives the parents information that is clear and concise.

Which child is in the greatest need of emergency medical treatment? 1. 3-year-old who has a barky cough, is afebrile, and has mild intercostal retractions. 2. 6-year-old who has high fever, no spontaneous cough, and frog-like croaking. 3. 7-year-old who has abrupt onset of moderate respiratory distress, a mild fever, and a barky cough. 4. 13-year-old who has a high fever, stridor, and purulent secretions

2. This child has signs and symptoms of epiglottitis and should receive immediate emergency medical treatment. The patient has no spontaneous cough and has a frog-like croaking because of a significant airway obstruction. TEST-TAKING HINT: The test taker must accurately identify that the question is describing a child with epiglottitis. The test taker must also understand that epiglottitis is a pediatric emergency and can cause the child to have complete airway obstruction.

Which assessment is of greatest concern in a 15-month-old? 1. The child is lying down, has moderate retractions, low-grade fever, and nasal congestion. 2. The child is in the tripod position, has diminished breath sounds, and a muffled cough. 3. The child is sitting up and has coarse breath sounds, coughing, and fussiness. 4. The child is restless and crying, has bilateral wheezes, and is feeding poorly.

2. When children are sitting in the tripod position, they are having difficulty breathing. The child is sitting and leaning forward in order to breathe more easily. Diminished breath sounds are indicative of a worsening condition. A muffled cough indicates that the child has some subglottic edema. This child has several signs and symptoms of a worsening respiratory condition. TEST-TAKING HINT: The test taker can eliminate answers 1, 3, and 4 if familiar with common signs and symptoms of respiratory illness.

What information should the nurse provide the parent of a child diagnosed with nasopharyngitis? 1. Complete the entire prescription of antibiotics. 2. Avoid sending the child to day care. 3. Use comfort measures for the child. 4. Restrict the child to clear liquids for 24 hours.

3. Nursing care for nasopharyngitis is primarily supportive. Keeping the child comfortable during the course of the illness is all the parents can do. Nasal congestion can be relieved using normal saline drops and bulb suction. Tylenol can also be given for discomfort or a mild fever. TEST-TAKING HINT: This question requires the test taker to understand how nasopharyngitis is treated.

An infant is scheduled for a hypospadias and chordee repair. The parent tells the nurse, "I understand why the hypospadias repair is necessary, but do they have to fix the chordee as well?" Which is the nurse's best response? 1. "I understand your concern. Parents do not want their children to undergo extra surgery." 2. "The chordee repair is done strictly for cosmetic reasons that may affect your son as he ages." 3. "The repair is done to optimize sexual functioning when he is older." 4. "This is the best time to repair the chordee because he will be having surgery anyway."

3. Releasing the chordee surgically is necessary for future sexual function.

Which statement by the parents of a toddler with repeated otitis media indicates they need additional teaching? 1. "If I quit smoking, my child may have a decreased chance of getting an ear infection." 2. "As my child gets older, he should have fewer ear infections, because his immune system will be more developed." 3. "My child will have fewer ear infections if he has his tonsils removed." 4. "My child may need a speech evaluation.

3. Removing children's tonsils may not have any effect on their ear infection. Children who have repeated bouts of tonsillitis can have ear infections secondary to the tonsillitis, but there is no indication in this question that the child has a problem with tonsillitis. TEST-TAKING HINT: The test taker can eliminate answers 1, 2, and 4 because those options are true.

A renal transplantation is which of the following? 1. A curative procedure that will free the child from any more treatment modalities. 2. An ideal treatment option for families with a history of dialysis noncompliance. 3. A treatment option that will free the child from dialysis. 4. A treatment option that is very new to the pediatric population.

3. Renal transplantation frees the patient from dialysis.

The nurse is giving discharge instructions to the parent of a 1-month-old infant with tracheoesophageal fistula and a gastrostomy tube (GT). The nurse knows the mother understands the discharge teaching when she states: 1. "I will give my baby feedings through the GT but place liquid medications in the corner of the mouth to be absorbed." 2. "I will flush the GT with 2 ounces of water after each feeding to prevent the GT from clogging." 3. "I will clean the area around the GT with soap and water every day." 4. "I will place petroleum jelly around the GT if any redness develops."

3. The area around the GT should be cleaned with soap and water to prevent an infection.

Which would the nurse expect to find on assessment in a child with Wilms tumor? 1. Decreased blood pressure, increased temperature, and a firm mass located in one flank area. 2. Increased blood pressure, normal temperature, and a firm mass located in one flank area. 3. Increased blood pressure, normal temperature, and a firm mass located on one side of the midline of the abdomen. 4. Decreased blood pressure, normal temperature, and a firm mass located on one side or the other of the midline of the abdomen.

3. The blood pressure may be increased if there is renal damage. The mass will be located on one side or the other of the midline of the abdomen. There is no reason for the child's temperature to be affected.

An adolescent woke up complaining of intense pain and swelling of the scrotal area and abdominal pain. He has vomited twice. Which should the nurse suggest? 1. Encourage him to drink clear liquids until the vomiting subsides; if he gets worse, bring him to the emergency room. 2. Bring him to the pediatrician's office for evaluation. 3. Take him to the emergency room immediately. 4. Encourage him to rest; apply ice to the scrotal area, and go to the emergency room if the pain does not improve.

3. The child is having symptoms of testicular torsion, which is a surgical emergency and needs immediate attention.

A parent asks the nurse how it will be determined if their child has respiratory syncytial virus (RSV). Which is the nurse's best response? 1. "We will do a simple blood test to determine whether your child has RSV." 2. "There is no specific test for RSV. The diagnosis is made based on the child's symptoms." 3. "We will swab your child's nose and send that specimen for testing." 4. "We will have to send a viral culture to an outside lab for testing."

3. The child is swabbed for nasal secretions. The secretions are tested to determine if a child has RSV. TEST-TAKING HINT: The test taker can eliminate answers 1, 2, and 4 because the child's nasal sections will be swabbed.

The diet for a child with chronic renal failure (CRF) should be high in calories and include: 1. Low protein, and all minerals and electrolytes. 2. Low protein and minerals. 3. High protein and calcium and low potassium and phosphorus. 4. High protein, phosphorus, and calcium and low potassium and sodium.

3. The child with CRF needs a diet high in calories, protein, and calcium and low in potassium and phosphorus.

The nurse is caring for a newborn with an anorectal malformation and a colostomy. The nurse knows that more education is needed when the infant's parent states which of the following? 1. "I will make sure the stoma is red." 2. "There should not be any discharge or irritation around the outside of the stoma." 3. "I will keep a bag attached to avoid the contents of the small intestine coming in contact with the baby's skin." 4. "As my baby grows, a pattern will develop over time, and there should be predictable bowel movements."

3. The colostomy contains stool from the large intestine; an ileostomy contains the very irritating stool from the small intestine.

Which is the best position for an 8-year-old who has just returned to the pediatric unit after an appendectomy for a ruptured appendix? 1. Semi-Fowler. 2. Prone. 3. Right side-lying. 4. Left side-lying.

3. The right side-lying position promotes comfort and allows the peritoneal cavity to drain.

The mother of a newborn asks the nurse why the infant has to nurse so frequently. Which is the best response? 1. Formula tends to be more calorically dense, and formula-fed babies require fewer feedings than breastfed babies. 2. The newborn's stomach capacity is small, and peristalsis is slow. 3. The newborn's stomach capacity is small, and peristalsis is more rapid than in older children 4. breastfed babies tend to take longer to complete a feeding than formula-fed babies.

3. The small-stomach capacity and rapid movement of fluid through the digestive system account for the need for small frequent feedings.

The nurse is caring for a 4-month-old who has just had an isolated cleft lip repaired. Select the best position for the child in the immediate post-operative period. 1. Right side-lying. 2. Left side-lying. 3. Supine. 4. Prone.

3. The supine position is preferred because there is decreased risk of the infant rubbing the suture line.

Which child diagnosed with pneumonia would benefit most from hospitalization? 1. 13-year-old who is coughing, has coarse breath sounds, and is not sleeping well 2. 14-year-old with a fever of 38.6°C (101.5°F), rapid breathing, and a decreased appetite. 3. 15-year-old who has been vomiting for 3 days and has a fever of 38.5°C (101.3°F). 4. A 16-year-old who has a cough, chills, fever of 38.5°C (101.3°F), and wheezing.

3. The teen who has been vomiting for several days and is unable to tolerate oral fluids and medication should be admitted for intravenous hydration. TEST-TAKING HINT: The test taker can eliminate answers 1, 2, and 4 if familiar with the common signs and symptoms of pneumonia.

A child diagnosed with a Wilms tumor is scheduled for an MRI scan of the lungs. The parent asks the nurse the reason for this test as a Wilms tumor involves the kidney, not the lung. Which is the nurse's best response? 1. "I'm not sure why your child is going for this test. I will check and get back to you." 2. "It sounds like we made a mistake. I will check and get back to you." 3. "The test is done to check to see if the disease has spread to the lungs." 4. "We want to check the lungs to make sure your child is healthy enough to tolerate surgery."

3. The test is done to see if the disease has spread to the lungs.

The nurse is caring for a 3-month-old being evaluated for possible Hirschsprung disease. His parents call the nurse and show her his diaper containing a large amount of mucus and bloody diarrhea. The nurse notes that the infant is irritable and his abdomen appears very distended. Which should be the nurse's next action? 1. Reassure the parents that this is an expected finding and not uncommon. 2. Call a code for a potential cardiac arrest, and stay with the infant. 3. Immediately obtain all vital signs with a quick head-to-toe assessment. 4. Obtain a stool sample for occult blood.

3. All vital signs need to be evaluated because the child with enterocolitis can quickly progress to a state of shock. A quick head-to-toe assessment will allow the nurse to evaluate the child's circulatory system

Which child does not need a urinalysis to evaluate for a urinary tract infection (UTI)? 1. A 4-month-old female presenting with a 2-day history of fussiness and poor appetite; current vital signs include axillary T 100.8°F (38.2°C), HR 120 beats per minute. 2. A 4-year-old female who states, "It hurts when I pee"; she has been urinating every 30 minutes; vital signs are within normal range. 3. An 8-year-old male presenting with a finger laceration; mother states he had surgical reimplantation of his ureters 2 years ago. 4. A 12-year-old female complaining of pain to her lower right back; she denies any burning or frequency at this time; oral temperature of 101.5°F (38.6°C).

3. Although this child has had a history of urinary infections, the child is currently not displaying any signs and therefore does not need a urinalysis at this time.

Which breathing exercises should the nurse have an asthmatic 3-year-old child do to increase her expiratory phase? 1. Use an incentive spirometer. 2. Breathe into a paper bag. 3. Blow a pinwheel. 4. Take several deep breaths.

3. Blowing a pinwheel is an excellent means of increasing a child's expiratory phase. Play is an effective means of engaging a child in therapeutic activities. Blowing bubbles is another method to increase the child's expiratory phase. TEST-TAKING HINT: The test taker can eliminate answers 1 and 4 because they do not increase the expiratory phase. Play is one of the best ways to engage young children in therapeutic activities.

The nurse is caring for a 9-month-old with diarrhea secondary to rotavirus. The child has not vomited and is mildly dehydrated. Which is likely to be included in the discharge teaching? 1. Administer Imodium as needed. 2. Administer Kaopectate as needed. 3. Continue breastfeeding per routine. 4. The infant may return to day care 24 hours after antibiotics have been started.

3. Breastfeeding is usually well tolerated and helps prevent death of intestinal villi and malabsorption.

A child diagnosed with acute renal failure (ARF) complains of "not feeling well," having "butterflies in the chest," and arms and legs "feeling like Jell-O." The cardiac monitor shows that the QRS complex is wider than before and that an occasional premature ventricular contraction (PVC) is seen. Which would the nurse expect to administer? 1. An isotonic saline solution with 20 mEq KCl/L. 2. Sodium bicarbonate via slow intravenous push. 3. Calcium gluconate via slow intravenous push. 4. Oral potassium supplements.

3. Calcium gluconate is the drug of choice for cardiac irritability secondary to hyperkalemia.

A 5-year-old is discharged from the hospital following the diagnosis of hemolytic uremic syndrome (HUS). The child has been free of diarrhea for 1 week, and renal function has returned. The parent asks the nurse when the child can return to school. Which is the nurse's best response? 1. "Immediately, as your child is no longer contagious." 2. "It would be best to keep your child home for a few more weeks because the immune system is weak, and there could be a relapse of HUS." 3. "Your child will be contagious for approximately another 10 days, so it is best to not allow a return just yet." 4. "It would be best to keep your child home to monitor urinary output."

3. Children with HUS are considered contagious for up to 17 days after the resolution of diarrhea and should be placed on contact isolation.

Which laboratory results besides hematuria are most consistent with hemolytic uremic syndrome (HUS)? 1. Massive proteinuria, elevated blood urea nitrogen, and creatinine. 2. Mild proteinuria, decreased blood urea nitrogen, and creatinine. 3. Mild proteinuria, increased blood urea nitrogen, and creatinine. 4. Massive proteinuria, decreased blood urea nitrogen, and creatinine.

3. Hematuria, mild proteinuria, increased BUN, and creatinine are all present in HUS.

The parents of a child hospitalized with minimal change nephrotic syndrome (MCNS) ask why the last blood test revealed elevated lipids. Which is the nurse's best response? 1. "If your child had just eaten a fatty meal, the lipids may have been falsely elevated." 2. "It's not unusual to see elevated lipids in children because of the dietary habits of today." 3. "Since your child is losing so much protein, the liver is stimulated and makes more lipids." 4. "Your child's blood is very concentrated because of the edema, so the lipids are falsely elevated."

3. In MCNS, the lipids are truly elevated. Lipoprotein production is increased because of the increased stimulation of the liver hypoalbuminemia.

A child with minimal change nephrotic syndrome (MCNS) has generalized edema. The skin appears stretched, and areas of breakdown are noted over the bony promi- nences. The child has been receiving Lasix twice daily for several days. Which does the nurse expect to be included in the treatment plan to reduce edema? 1. An increase in the amount and frequency of Lasix. 2. Addition of a second diuretic, such as mannitol. 3. Administration of intravenous albumin. 4. Elimination of all fluids and sodium from the child's diet.

3. In cases of severe edema, albumin is used to help return the fluid to the bloodstream from the subcutaneous tissue.

The nurse is caring for a 1-year-old diagnosed with acute renal failure (ARF). Edema is noted throughout the child's body, and the liver is enlarged. The child's urine out- put is less than 0.5 mL/kg/hr, and vital signs are as follows: HR 146, BP 176/92, and RR 42. The child is noted to have nasal flaring and retractions with inspiration. The lung sounds are coarse throughout. Despite receiving oral Kayexalate, the child's serum potassium continues to rise. Which treatment will provide the most benefit to the child? 1. Additional rectal Kayexalate. 2. Intravenous furosemide. 3. Endotracheal intubation and ventilatory assistance. 4. Placement of a Tenckhoff catheter for peritoneal dialysis.

4. Placement of a Tenckhoff catheter for peritoneal dialysis is needed when the child's condition deteriorates despite medical treatment.

The parent of a newborn asks, "Will my baby spit out the formula if it is too hot or too cold?" Select the nurse's best response. 1. "Babies have a tendency to reject hot fluids but not cold fluids, which could result in abdominal discomfort." 2. "Babies have a tendency to reject cold fluids but not hot fluids, which could result in esophageal burns." 3. "Your baby would most likely spit out formula that was too hot, but your baby could swallow some of it, which could result in a burn." 4. "Your baby is too young to be physically capable of spitting out fluids and will automatically swallow anything."

4. Swallowing is a reflex in infants younger than 6 weeks.

The nurse is caring for an 8-week-old male who has just been diagnosed with Hirschsprung disease. The parents ask what they should expect. Select the nurse's best response. 1. "It is really an easy disease to manage. Most children are placed on stool softeners to help with constipation until it resolves." 2. "A permanent stool diversion, called a colostomy, will be placed by the surgeon to bypass the narrowed area." 3. "Daily bowel irrigations will help your child maintain regular bowel habits." 4. "Although your child will require surgery, there are different ways to manage the disease, depending on how much bowel is involved."

4. The aganglionic portion needs to be removed. Although most children have a temporary colostomy placed, many infants are able to bypass the colostomy and have the bowel immediately reattached.

In addition to increased blood pressure, which findings would most likely be found in a child with hydronephrosis? 1. Metabolic alkalosis, polydipsia, and polyuria. 2. Metabolic acidosis, and bacterial growth in the urine. 3. Metabolic alkalosis, and bacterial growth in the urine. 4. Metabolic acidosis, polydipsia, and polyuria.

4. The blood pressure is increased as the body attempts to compensate for the decreased glomerular filtration rate. Metabolic acidosis is caused by a reduction in hydrogen ion secretion from the distal nephron. Polydipsia and polyuria occur as the kidney's ability to concentrate urine decreases. There is bacterial growth in the urine due to the urinary stasis caused by the obstruction.

The bladder capacity of a 3-year-old is approximately how much? 1. 1.5 fl. oz. 2. 3 fl. oz. 3. 4 fl. oz. 4. 5 fl. oz.

4. The capacity of the bladder in fluid ounces can be estimated by adding 2 to the child's age in years.

The nurse evaluates post-operative teaching for repair of testicular torsion as successful when the parent of an adolescent says which the following? 1. "I will encourage him to rest for a few days, but he can return to football practice in a week." 2. "I will keep him in bed for 4 days and let him gradually increase his activity after that." 3. "I will seek therapy as he ages because he is now infertile." 4. "I will make sure he does testicular self-examination monthly."

4. The child and family should be taught the importance of testicular self-examination.

The parent of a 9-month-old calls the ER because his child is choking on a marble. The parent asks how to help his child while awaiting Emergency Medical Services. Which is the nurse's best response? 1. "You should administer five abdominal thrusts followed by five back blows." 2. "You should try to retrieve the object by inserting your finger in your child's mouth." 3. "You should perform the Heimlich maneuver." 4. "You should administer five back blows followed by five chest thrusts."

4. The current recommendation for infants younger than 1 year is to administer five back blows followed by five chest thrusts. TEST-TAKING HINT: The test taker can eliminate answers 1, 2, and 3 if familiar with CPR in infants and children.

The nurse evaluates the parents' understanding of the teaching about an inguinal hernia as successful when they say which of the following? 1. "There are no risks associated with waiting to have the hernia reduced; surgery is done for cosmetic reasons." 2. "It is normal to see the bulge in the baby's groin decrease with a bowel movement." 3. "We will wait for surgery until the baby is older because narcotics for pain control will be required for several days." 4. "It is normal for the bulge in the baby's groin to look smaller when the baby is asleep."

4. The hernia often appears smaller when the child is asleep.

Which should be the nurse's immediate action when a newborn begins to cough and choke and becomes cyanotic while feeding? 1. Inform the physician of the situation. 2. Have the mother stop feeding the infant, and observe to see if the choking episode resolves on its own. 3. Immediately determine the infant's oxygen saturation, and have the mother stop feeding the infant. 4. Take the infant from the mother, and administer blow-by oxygen while obtaining the infant's oxygen saturation.

4. The infant should be taken from the mother and placed in the crib where the nurse can assess the baby. Oxygen should be administered immediately, and vital signs should be obtained.

The parents of a newborn diagnosed with a cleft lip and palate ask the nurse when their child's lip and palate will most likely be repaired. Select the nurse's best response. 1. "The palate and the lip are usually repaired in the first few weeks of life so that the baby can grow and gain weight." 2. "The palate and the lip are usually not repaired until the baby is approximately 6 months old so that the mouth has had enough time to grow." 3. "The lip is repaired in the first few months of life, but the palate is not usually repaired until the child is 3 years old." 4. "The lip is repaired in the first few weeks of life, but the palate is not usually repaired until the child is 18 months old."

4. The lip is repaired in the first few weeks of life, but the palate is not usually repaired until the child is 18 months old.

A child with severe cerebral palsy is admitted to the hospital with aspiration pneumonia. What is the most beneficial educational information that the nurse can provide to the parents? 1. The signs and symptoms of aspiration pneumonia. 2. The treatment plan for aspiration pneumonia. 3. The risks associated with recurrent aspiration pneumonia. 4. The prevention of aspiration pneumonia.

4. The most valuable information the nurse can give the parents is how to prevent aspiration pneumonia from occurring in the future. TEST-TAKING HINT: The test taker can eliminate answers 1, 2, and 3 because they are all forms of tertiary prevention. Primary prevention is key to maximizing this child's function.

A mother is crying and tells the nurse that she should have brought her son in yesterday when he said his throat was sore. Which is the nurse's best response to this parent whose child is diagnosed with epiglottitis and is in severe distress and in need of intubation? 1. "Children this age rarely get epiglottitis; you should not blame yourself." 2. "It is always better to have your child evaluated at the first sign of illness rather than wait until symptoms worsen." 3. "Epiglottitis is slowly progressive, so early intervention may have decreased the extent of your son's symptoms." 4. "Epiglottitis is rapidly progressive; you could not have predicted his symptoms would worsen so quickly."

4. Epiglottitis is rapidly progressive and cannot be predicted. TEST-TAKING HINT: When something happens to a child, the parents always blame themselves. Telling them epiglottitis is rapidly progressive may be helpful.

A school-age child is admitted to the hospital for a tonsillectomy. During the nurse's post-operative assessment, the child's parent tells the nurse that the child is in pain. Which of the following observations would be of most concern to the nurse? 1. The child's heart rate and blood pressure are elevated. 2. The child complains of having a sore throat. 3. The child is refusing to eat solid foods. 4. The child is swallowing excessively.

4. Excessive swallowing is a sign that the child is swallowing blood. This should be considered a medical emergency, and the physician should be contacted immediately. The child is likely bleeding and will need to return to surgery. TEST-TAKING HINT: Answer 1 can be eliminated if the test taker understands the common vital-sign changes that occur when a person is experiencing pain.

A parent asks the nurse how to prevent the child from having minimal change nephrotic syndrome (MCNS) again. Which is the nurse's best response? 1. "It is very rare for a child to have a relapse after having fully recovered." 2. "Unfortunately, many children have cycles of relapses, and there is very little that can be done to prevent it." 3. "Your child is much less likely to get sick again if sodium is decreased in the diet." 4. "Try to keep your child away from sick children because relapses have been associated with infectious illnesses."

4. Exposure to infectious illness has been linked to the relapse of nephrotic syndrome.

Which should the nurse teach a group of girls and parents about the importance of preventing urinary tract infections (UTIs)? 1. Avoiding constipation has no effect on the occurrence of UTIs. 2. After urinating, always wipe from back to front to prevent fecal contamination. 3. Hygiene is an important preventive measure and can be accomplished with frequent tub baths. 4. Increasing fluids will help prevent and treat UTIs.

4. Increasing fluids will help flush the bladder of any organism, encourage urination, and prevent stasis of urine.

The nurse will soon receive a 4-month-old who has been diagnosed with intussuscep- tion. The infant is described as very lethargic with the following vital signs: T 101.8°F (38.7°C), HR 181, BP 68/38. The reporting nurse states the infant's abdomen is very rigid. Which is the most appropriate action for the receiving nurse? 1. Prepare to accompany the infant to a computed tomography scan to confirm the diagnosis. 2. Prepare to accompany the infant to the radiology department for a reducing enema. 3. Prepare to start a second intravenous line to administer fluids and antibiotics. 4. Prepare to get the infant ready for immediate surgical correction.

4. Intussusception with peritonitis is a surgical emergency, so preparing the infant for surgery is the nurse's top priority.

The parents of a 6-year-old who has a new diagnosis of asthma ask the nurse what to do to make their home a more allergy-free environment. Which is the nurse's best response? 1. "Use a humidifier in your child's room." 2. "Have your carpet cleaned chemically once a month." 3. "Wash household pets weekly." 4. "Avoid purchasing upholstered furniture."

4. Leather furniture is recommended rather than upholstered furniture. Upholstered furniture can harbor large amounts of dust, whereas leather furniture may be wiped off regularly with a damp cloth. TEST-TAKING HINT: The test taker can eliminate answer 3 because there is no known way to make a pet allergy-free. Household pets are discouraged for all children with asthma or severe allergies. Answer 2 can be eliminated if the test taker understands that chemical agents are triggers to asthma for many children.

The nurse receives a call from the parent of a 10-month-old who has vomited three times in the past 8 hours. The parent describes the baby as playful and wanting to drink. The parent asks the nurse what to give the child. Select the nurse's best response. 1. "Replace the next feeding with regular water, and see if that is better tolerated." 2. "Do not allow your baby to eat any solids; give half the normal formula feeding, and see if that is better tolerated." 3. "Do not let your baby eat or drink anything for 24 hours to give the stomach a chance to rest." 4. "Give your child 1 /2 ounce of Pedialyte every 10 minutes. If vomiting continues, wait an hour, and then repeat what you previously gave."

4. Offering small amounts of clear liquids is usually well tolerated. If the child vomits, make NPO to allow the stom- ach to rest and then restart fluids. The child in this scenario is described as playful and therefore does not appear to be at risk for dehydration.

Which medication would most likely be included in the post-operative care of a child with repair of bladder exstrophy? 1. Lasix. 2. Mannitol. 3. Meperidine. 4. Oxybutynin.

4. Oxybutynin is used to control bladder spasms.

Which manifestation would the nurse expect to see in a 4-week-old infant with biliary atresia? 1. Abdominal distention, enlarged liver, enlarged spleen, clay-colored stool, and tea-colored urine. 2. Abdominal distention, multiple bruises, bloody stools, and hematuria. 3. Yellow sclera and skin tones, excessively oily skin, and prolonged bleeding times. 4. No manifestations until the disease has progressed to the advanced stage.

1. The infant with biliary atresia usually has an enlarged liver and spleen. The stools appear clay-colored due to the absence of bile pigments. The urine is tea-colored due to the excretion of bile salts.

Which should be included in the plan of care for a 14-month-old whose cleft palate was repaired 12 hours ago? Select all that apply. 1. Allow the infant to have familiar items of comfort such as a favorite stuffed animal and a "sippy" cup. 2. Once liquids have been tolerated, encourage a bland diet such as soup, Jell-O, and saltine crackers. 3. Administer pain medication on a regular schedule, as opposed to an as-needed schedule. 4. Use a Yankauer suction catheter on the infant's mouth to decrease the risk of aspiration of oral secretions. 5. When discharged, remove elbow restraints.

1, 3. 1. The child should not be allowed to use anything that creates suction in the mouth, such as pacifiers or straws. "Sippy" cups are acceptable. 2. The child should not have anything hard in the mouth, such as crackers, cookies, or a spoon. 3. Pain medication should be adminis- tered regularly to avoid crying, which places stress on the suture line. 4. A Yankauer suction should not be used in the mouth because it creates suction and is a hard instrument that could irritate the suture line. The child should be positioned to allow secretions to drain out of the child's mouth. Suction should be used only in the event of an emergency. 5. Elbow restraints are used until the repaired palate has healed. When at home, the par- ents need to monitor the child closely if restraints are removed to move the arms or for bathing. TEST-TAKING HINT: The child who has had a cleft palate repair should have nothing in the mouth that could irritate the suture line. Answers 2 and 4 can be eliminated.

Which foods should be offered to a child with hepatitis? 1. A tuna sandwich on whole wheat bread and a cup of skim milk. 2. Clear liquids, such as broth, and Jell-O. 3. A hamburger, French fries, and a diet soda. 4. A peanut butter sandwich and a milkshake.

1. A diet that is high in protein and carbohydrates helps maintain caloric intake and protein stores while preventing muscle wasting. A low-fat diet prevents abdominal distention.

A 4-month-old is brought to the emergency department with severe dehydration. The heart rate is 198, and her blood pressure is 68/38. The infant's anterior fontanel is sunken. The nurse notes that the infant does not cry when the intravenous line is inserted. The child's parents state that she has not "held anything down" in 18 hours. The nurse obtains a finger-stick blood sugar of 94. Which would the nurse expect to do immediately? 1. Administer a bolus of normal saline. 2. Administer a bolus of D10W. 3. Administer a bolus of normal saline with 5% dextrose added to the solution. 4. Offer the child an oral rehydrating solution such as Pedialyte.

1. Dehydration is corrected with the administration of an isotonic solution, such as normal saline or lactated Ringer solution.

Which child is at highest risk for requiring hospitalization to treat respiratory syncytial virus (RSV)? 1. A 2-month-old who was born at 32 weeks. 2. A 16-month-old with a tracheostomy. 3. A 3-year-old with a congenital heart defect. 4. A 4-year-old who was born at 30 weeks.

1. The younger the child, the greater the risk for developing complications related to RSV. This infant is at highest risk because of age and premature status. TEST-TAKING HINT: The test taker must consider that all of these children have some amount of risk for requiring hospitalization. The 2-month-old has two of the noted risk factors of being premature and a very young infant.

...

...

Which discharge instruction for a child diagnosed with encopresis should the nurse question? 1. Limit the intake of milk. 2. Offer a diet high in protein. 3. Obtain a complete dietary log. 4. Follow up with a child psychologist.

2. A diet high in protein will cause more constipation.

Which child would benefit most from having ear tubes placed? 1. A 2-month-old who has had one ear infection. 2. A 2-year-old who has had five previous ear infections. 3. A 3-year-old whose sibling has had four ear infections. 4. A 7-year-old who has had two ear infections this year.

2. A 2-year-old who has had multiple ear infections is a perfect candidate for ear tubes. The other issue is that a 2-year-old is at the height of language development, which can be adversely affected by recurrent ear infections. TEST-TAKING HINT: The test taker must also consider the developmental level of the child in this question. The 2-year-old has had multiple infections and is also at a stage when language development is essential. If this child is not hearing appropriately, speech will also be delayed. Surgical intervention is reserved for those who have had recurrent infections.

The parents of a child being evaluated for appendicitis tell the nurse the physician said their child has a positive Rovsing sign. They ask the nurse what this means. Select the nurse's best response. 1. "Your child's physician should answer that question." 2. "A positive Rovsing sign means the child feels pain in the right side of the abdomen when the left side is palpated." 3. "A positive Rovsing sign means pain is felt when the physician removes the hand from the abdomen." 4. "A positive Rovsing sign means pain is felt in the right lower quadrant when the child coughs."

2. A positive Rovsing sign occurs when the left lower quadrant is palpated and pain is felt in the right lower quadrant.

A school-age child has been diagnosed with strep throat. The parent asks the nurse when the child can return to school. Which is the nurse's best response? 1. "Forty-eight hours after the first documented normal temperature." 2. "Twenty-four hours after the first dose of antibiotics." 3. "Forty-eight hours after the first dose of antibiotics." 4. "Twenty-four hours after the first documented normal temperature."

2. Children with strep throat are no longer contagious 24 hours after initiation of antibiotic therapy. TEST-TAKING HINT: The test taker can eliminate answers 1 and 4 given knowledge of the communicability of strep throat.

How does the nurse interpret the laboratory analysis of a stool sample containing excessive amounts of azotorrhea and steatorrhea in a child with cystic fibrosis (CF)? The values indicate the child is 1. Not compliant with taking her vitamins. 2. Not compliant with taking her enzymes. 3. Eating too many foods high in fat. 4. Eating too many foods high in fiber.

2. If the child were not taking enzymes, the result would be a large amount of undigested food, azotorrhea, and steatorrhea in the stool. Pancreatic ducts in CF patients become clogged with thick mucus that blocks the flow of digestive enzymes from the pancreas to the duodenum. Therefore, patients must take digestive enzymes with all meals and snacks to aid in absorption of nutrients. Often, teens are noncompliant with their medication regimen because they want to be like their peers TEST-TAKING HINT: The test taker needs to understand the pathophysiology of CF and the impact it has on the gastrointestinal system. The test taker also must be familiar with the conditions azotorrhea and steatorrhea.

A 6-week-old is admitted to the hospital with influenza. The child is crying, and the father tells the nurse that his son is hungry. The nurse explains that the baby is not to have anything by mouth. The parent does not understand why the child cannot eat. Which is the nurse's best response to the parent? 1. "We are giving your child intravenous fluids, so there is no need for anything by mouth." 2. "The shorter and narrower airway of infants increases their chances of aspiration so your child should not have anything to eat now." 3. "When your child eats, he burns too many calories; we want to conserve the child's energy." 4. "Your child has too much nasal congestion; if we feed the child by mouth, the distress will likely increase."

2. Infants are at higher risk of aspiration because their airways are shorter and narrower than those of adults. An infant with influenza has lots of nasal secretions and coughs up mucus. With all the secretions, the infant is at an even higher risk of aspiration. TEST-TAKING HINT: The test taker can eliminate answer 1 because it does not give the father an explanation of why his son cannot eat.

Which child can be discharged without further evaluation? 1. A 2-year-old who has had 24 hours of watery diarrhea that has changed to bloody diarrhea in the past 12 hours. 2. A 2-year-old who had a relapse of one diarrhea episode after restarting a normal diet. 3. A 6-year-old who has been having vomiting and diarrhea for 2 days and has decreased urine output. 4. A 10-year-old who has just returned from a Scout camping trip and has had several episodes of diarrhea.

2. It is common for children to have a relapse of diarrhea after resuming a regular diet.

The nurse is caring for a newborn with a cleft lip and palate. The mother states, "I will not be able to breastfeed my baby." Select the nurse's best response. 1. "It sounds like you are feeling discouraged. Would you like to talk about it?" 2. "Sometimes breastfeeding is still an option for babies with a cleft lip and palate. Would you like more information?" 3. "Although breastfeeding is not an option, you have the option of pumping your milk and then feeding it to your baby with a special nipple." 4. "We usually discourage breastfeeding babies with cleft lip and palate as it puts them at an increased risk for aspiration."

2. Some mothers are able to breastfeed their infants who have a cleft lip and palate. The breast can help fill in the cleft and help the infant create suction.

Which would be an early sign of respiratory distress in a 2-month-old? 1. Breathing shallowly. 2. Tachypnea. 3. Tachycardia. 4. Bradycardia.

2. Tachypnea is an early sign of distress and is often the first sign of respiratory illness in infants TEST-TAKING HINT: The test taker must know the signs and symptoms of respiratory distress and be able to recognize them

Which laboratory result will provide the most important information regarding the respiratory status of a child with an acute asthma exacerbation? 1. CBC. 2. ABG. 3. BUN. 4. PTT.

2. The ABG gives the health-care team valuable information about the child's respiratory status: level of oxygenation, carbon dioxide, and blood pH. TEST-TAKING HINT: The test taker can eliminate answers 1, 2 and 3 with a knowledge of common laboratory tests.

The nurse knows that Nissen fundoplication involves which of the following? 1. The fundus of the stomach is wrapped around the inferior stomach, mimicking a lower esophageal sphincter. 2. The fundus of the stomach is wrapped around the inferior esophagus, mimicking a cardiac sphincter. 3. The fundus of the stomach is wrapped around the middle portion of the stomach, decreasing the capacity of the stomach. 4. The fundus of the stomach is dilated, decreasing the likelihood of reflux.

2. The Nissen fundoplication involves wrapping the fundus of the stomach around the inferior esophagus, creat- ing a lower esophageal sphincter or cardiac sphincter.

The parent of a child with frequent ear infections asks the nurse if there is anything that can be done to help avoid future ear infections. Which is the nurse's best response? 1. "Your child should be put on a daily dose of Singulair (montelukast)." 2. "Your child should be kept away from tobacco smoke." 3. "Your child should be kept away from other children with otitis media." 4. "Your child should always wear a hat when outside."

2. Tobacco smoke has been proved to increase the incidence of ear infections. The tobacco smoke damages mucociliary function, prolonging the inflammatory process and impeding drainage through the eustachian tube. TEST-TAKING HINT: The test taker can eliminate answer 3 by understanding that otitis media is not a contagious disease process. Answer 4 can be eliminated if the test taker understands that otitis media is not caused by exposing the child to cold air.

Which is the nurse's best response to parents who ask what impact asthma will have on the child's future in sports? 1. "As long as your child takes prescribed asthma medication, the child will be fine." 2. "The earlier a child is diagnosed with asthma, the more significant the symptoms." 3. "The earlier a child is diagnosed with asthma, the better the chance the child has of growing out of the disease." 4. "Your child should avoid playing contact sports and sports that require a lot of running."

2. When a child is diagnosed with asthma at an early age, the child is more likely to have significant symptoms on aging. TEST-TAKING HINT: The test taker can eliminate answer 4 because not all asthmatics also have exercise-induced asthma necessitating use of a fast-acting bronchodilator before playing.

Which would be appropriate nursing care management of a child with the diagnosis of mononucleosis? 1. Only family visitors. 2. Bedrest. 3. Clear liquids. 4. Limited daily fluid intake.

26. 1. Children with mononucleosis are more susceptible to secondary infections. Therefore, they should be limited to visitors within the family, especially during the acute phase of illness. TEST-TAKING HINT: The test taker can eliminate answers 2 and 3 by understanding mononucleosis. Children with mononucleosis are usually very tired, are not interested in engaging in vigorous activity, and are rarely interested in eating.

Which would be an appropriate activity for the nurse to recommend to the parent of a toddler just diagnosed with acute hepatitis? 1. Climbing in a "playscape." 2. Kicking a ball. 3. Playing video games in bed. 4. Playing with puzzles in bed.

4. Playing with puzzles is a developmen- tally appropriate activity for a 3-year-old on bedrest.

A parent asks the nurse what will need to be done to relieve the constipation of her child who also has cystic fibrosis (CF). Which is the nurse's best response? 1. "Your child likely has an obstruction and will require surgery." 2. "Your child will likely be given IV fluids." 3. "Your child will likely be given MiraLAX." 4. "Your child will be placed on a clear liquid diet."

3. CF patients with constipation commonly receive a stool softener or an osmotic solution such as polyethylene glycol 3350 (MiraLAX) orally to relieve their constipation. TEST-TAKING HINT: Answer 1 can be eliminated because surgery is not indicated for constipation.

The parent of an infant with cystic fibrosis (CF) asks the nurse how to meet the child's increased nutritional needs. Which is the nurse's best suggestion? 1. "You may need to increase the number of fresh fruits and vegetables you give your child." 2. "You may need to advance your child's diet to whole cow's milk because it is higher in fat than formula." 3. "You may need to change your child to a higher-calorie formula." 4. "You may need to increase your child's carbohydrate intake."

3. Often infants with CF need to have a higher-calorie formula to meet their nutritional needs. Infants may also be placed on hydrolysate formulas that contain medium-chain triglycerides. TEST-TAKING HINT: Answers 1, 2, and 4 can be eliminated with understanding of the nutritional needs of the child with CF. Answer 2 can also be eliminated because whole cow's milk is not recommended until 12 months of age.

Which information will be most helpful in teaching parents about the primary prevention of foreign body aspiration? 1. Signs and symptoms of foreign body aspiration. 2. Therapeutic management of foreign body aspiration. 3. Most common objects that toddlers aspirate. 4. Risks associated with foreign body aspiration.

3. Teaching parents the most common objects aspirated by toddlers will help them the most. Parents can avoid having those items in the household or in locations where toddlers may have access to them. TEST-TAKING HINT: The test taker can eliminate answers 1, 2, and 4 because they are all forms of tertiary prevention. Primary prevention is key to preventing foreign body aspiration.

A 4-month-old has had vomiting and diarrhea for 24 hours. The infant is fussy, and the anterior fontanel is sunken. The nurse notes the infant does not produce tears when crying. Which task will help confirm the diagnosis of dehydration? 1. Urinalysis obtained by bagged specimen. 2. Urinalysis obtained by sterile catheterization. 3. Analysis of serum electrolytes. 4. Analysis of cerebrospinal fluid.

3. The analysis of serum electrolytes offers the most information and assists with the diagnosis of dehydration.

A child's parent asks the nurse what treatment the child will need for the diagnosis of strep throat. Which is the nurse's best response? 1. "Your child will be sent home on bedrest and should recover in a few days without any intervention." 2. "Your child will need to have the tonsils removed to prevent future strep infections." 3. "Your child will need oral penicillin for 10 days and should feel better in a few days." 4. "Your child will need to be admitted to the hospital for 5 days of intravenous antibiotics."

3. The child will need a 10-day course of penicillin to treat the strep infection. It is essential that the nurse always tell the family that, although the child will feel better in a few days, the entire course of antibiotics must be completed. TEST-TAKING HINT: Answer 2 can be eliminated because it is a treatment for recurrent tonsillitis, not strep throat. Answer 1 can be eliminated if the test taker understands that bacterial infections need to be treated with antibiotics.

The nurse is caring for a 4-month-old with gastroesophageal reflux (GER). The infant is due to receive Zantac (rantadine). Based on the medication's mechanism of action, when should this medication be administered? 1. Immediately before a feeding. 2. 30 minutes after the feeding. 3. 30 minutes before the feeding. 4. At bedtime.

3. Zantac decreases gastric acid secretion and should be administered 30 minutes before a feeding.

Which should the nurse include in the plan of care to decrease symptoms of gastroesophageal reflux (GER) in a 2-month-old? Select all that apply. 1. Place the infant in an infant seat immediately after feedings. 2. Place the infant in the prone position immediately after feeding to decrease the risk of aspiration. 3. Encourage the parents not to worry because most infants outgrow GER within the first year of life. 4. Encourage the parents to hold the infant in an upright position for 30 minutes following a feeding. 5. Suggest that the parents burp the infant after every 1-2 ounces consumed.

4, 5. 1. Placing the infant in an infant seat increases intra-abdominal pressure, placing the infant at increased risk for GER. 2. The prone position is not recommended as it may lead to sudden infant death syndrome (SIDS). 3. Although most infants outgrow GER, providing the parents with this education will not help decrease the symptoms. 4. Keeping the infant in an upright posi- tion is the best way to decrease the symptoms of GER. The infant can also be placed in the supine position with the head of the crib elevated. A harness can be used to keep the child from sliding down. 5. Burping the infant frequently may help decrease spitting up by expelling air from the stomach more often.

Which physical findings would be of most concern in an infant with respiratory distress? 1. Tachypnea. 2. Mild retractions. 3. Wheezing. 4. Grunting.

4. Grunting is a sign of impending respiratory failure and is a very concerning physical finding. TEST-TAKING HINT: The test taker can eliminate answers 1, 2, and 3 by knowing the signs of respiratory distress. They warrant frequent respiratory assessment, but they are not the most concerning physical signs.

Which would the nurse explain to parents about the inheritance of cystic fibrosis? 1. CF is an autosomal-dominant trait passed on from the child's mother. 2. CF is an autosomal-dominant trait passed on from the child's father. 3. The child of parents who are both carriers of the gene for CF has a 50% chance of acquiring CF. 4. The child of a mother who has CF and a father who is a carrier of the gene for CF has a 50% chance of acquiring CF

4. If the child is born to a parent with CF and the other parent is a carrier, the child has a 50% chance of acquiring the disease and a 50% chance of being a carrier of the disease. TEST-TAKING HINT: Answers 1 and 2 can be eliminated with knowledge of the genetic inheritance of CF. CF is inherited as an autosomal-recessive trait.

The nurse is to receive a 4-year-old from the recovery room after an appendectomy. The parents have not seen the child since surgery and ask what to expect. Select the nurse's best response. 1. "Your child will be very sleepy, have an intravenous line in the hand, and have a nasal tube to help drain the stomach. If your child needs pain medication, it will be given intravenously." 2. "Your child will be very sleepy, have an intravenous line in the hand, and have white stockings to help prevent blood clots. If your child needs pain medication, we will give it intravenously or provide a liquid to swallow." 3. "Your child will be wide awake and will have an intravenous line in the hand. If your child needs pain medication, we will give it intravenously or provide a liquid to swallow." 4. "Your child will be very sleepy and have an intravenous line in the hand. If your child needs pain medication, we will give it intravenously."

4. In the immediate post-operative period, the child is usually sleepy but can be roused. The child usually has an intravenous line for hydration and pain medication.

A child is complaining of throat pain. Which statement by the mother indicates that she needs more education regarding the care and treatment of her daughter's pharyngitis? 1. "I will have my daughter gargle with salt water three times a day." 2. "I will offer my daughter ice chips several times a day." 3. "I will give my daughter Tylenol every 4 to 6 hours as needed." 4. "I will ask the nurse practitioner for some amoxicillin."

4. Pharyngitis is a self-limiting viral illness that does not require antibiotic therapy. Pharyngitis should be treated with rest and comfort measures, including Tylenol, throat sprays, cold liquids, and Popsicles. TEST-TAKING HINT: Answers 1, 2, and 3 are comfort measures. The question requires that the student have knowledge regarding pharyngitis.

What is the most important piece of information that the nurse must ask the parent of a child in status asthmaticus? 1. "What time did your child eat last?" 2. "Has your child been exposed to any of the usual asthma triggers?" 3. "When was your child last admitted to the hospital for asthma?" 4. "When was your child's last dose of medication?"

4. The nurse needs to know what medication the child had last and when the child took it in order to know how to begin treatment for the current asthmatic condition. TEST-TAKING HINT: Whereas all of the information here is essential, answer 4 gives the most important information. The test taker can eliminate answers 2 and 3 because the responses to these inquiries have no direct impact on the immediate treatment of the child. These two answers give information about the severity of the child's illness, but they do not affect the immediate treatment plan. Answer 4 is essential to deciding what medication should be given the child to relieve the current symptoms.

A 10-year-old is being evaluated for possible appendicitis and complains of nausea and sharp abdominal pain in the right lower quadrant. An abdominal ultrasound is scheduled, and a blood count has been obtained. The child vomits, finds the pain relieved, and calls the nurse. Which should be the nurse's next action? 1. Cancel the ultrasound, and obtain an order for oral Zofran (ondansetron). 2. Cancel the ultrasound, and prepare to administer an intravenous bolus. 3. Prepare for the probable discharge of the patient. 4. Immediately notify the physician of the child's status.

4. The physician should be notified immediately, as a sudden change or loss of pain often indicates a perforated appendix.

The parent of a child with croup tells the nurse that her other child just had croup and it cleared up in a couple of days without intervention. She asks the nurse why this child is exhibiting worse symptoms and needs to be hospitalized. Which is the nurse's best response? 1. "Some children just react differently to viruses. It is best to treat each child as an individual." 2. "Younger children have wider airways that make it easier for bacteria to enter and colonize." 3. "Younger children have short and wide eustachian tubes, making them more susceptible to respiratory infections." 4. "Children younger than 3 years usually exhibit worse symptoms because their immune systems are not as developed."

4. Younger children have less developed immune systems and usually exhibit worse symptoms than older children. TEST-TAKING HINT: Answer 1 can be eliminated because it does not directly address the mother's question. Answer 2 can be eliminated if the test taker has knowledge of the anatomical structure of a child's airway. Answer 3 can be eliminated because the eustachian tubes have no direct relationship to acquiring croup.

A chloride level greater than _____________________ is a positive diagnostic indicator of cystic fibrosis (CF).

60 mEq/L. The definitive diagnosis of CF is made when a child has a sweat chloride level >60 mEq/L. A normal chloride level is <40 mEq/L. TEST-TAKING HINT: The test taker must have knowledge of tests and normal values used to identify a diagnosis of CF.

Which child is at risk for developing glomerulonephritis? 1. A 3-year-old who had impetigo 1 week ago. 2. A 5-year-old with a history of five UTIs in the previous year. 3. A 6-year-old with new-onset type 1 diabetes. 4. A 10-year-old recovering from viral pneumonia.

1. Impetigo is a skin infection caused by the streptococcal organism that is commonly associated with glomerulonephritis.

Which would the nurse most likely find in the history of a child with hemolytic uremic syndrome (HUS)? 1. Frequent UTIs and possible vesicoureteral reflux (VUR). 2. Vomiting and diarrhea before admission. 3. Bee sting and localized edema of the site for 3 days. 4. Previously healthy and no signs of illness.

2. HUS is often preceded by diarrhea that may be caused by E. coli present in undercooked meat.

The parents of a 3-year-old are concerned that the child is having "more accidents" during the day. Which question would be appropriate for the nurse to ask to obtain more information? Select all that apply. 1. "Has there been a stressful event in the child's life, such as the birth of a sibling?" 2. "Has anyone else in the family had problems with accidents?" 3. "Does your child seem to be drinking more than usual?" 4. "Is your child more irritable, and does your child seem to be in pain when urinating?" 5. "Is your child having difficulties at preschool?"

1, 2, 3, 4. 1. Stressors such as the birth of a sibling can lead to incontinence in a child who previously had bladder control. 2. A pattern of enuresis can often be seen in families. 3. Increased thirst and incontinence can be associated with diabetes. 4. Irritability and incontinence can be associated with UTIs. 5. Preschool-age children do not habitually share, so this information would not help the nurse in gathering more information

61. Which child may need extra fluids to prevent dehydration? Select all that apply. 1. 7-day-old receiving phototherapy. 2. 6-month-old with newly diagnosed pyloric stenosis. 3. 2-year-old with pneumonia. 4. 2-year-old with full-thickness burns to the chest, back, and abdomen. 5. 13-year-old who has just started her menses.

1, 2, 3, 4. 1. The lights in phototherapy increase insensible fluid loss, requiring the nurse to monitor fluid status closely. 2. The infant with pyloric stenosis is likely to be dehydrated due to persistent vomiting. 3. A 2-year-old with pneumonia may have increased insensible fluid loss due to tachypnea associated with respira- tory illness. The nurse needs to moni- tor fluid status cautiously because fluid overload can result in increased respiratory distress. 4. The child with a burn experiences extensive extracellular fluid loss and is at great risk for dehydration. The younger child is at greater risk due to greater proportionate body surface area. 5. An adolescent starting her menses is not at risk for dehydration.

What does the therapeutic management of cystic fibrosis (CF) patients include? Select all that apply. 1. Providing a high-protein, high-calorie diet. 2. Providing a high-fat, high-carbohydrate diet. 3. Encouraging exercise. 4. Minimizing pulmonary complication. 5. Encouraging medication compliance.

1, 3, 4, 5. 1. Children with CF have difficulty absorbing nutrients because of the blockage of the pancreatic duct. Pancreatic enzymes cannot reach the duodenum to aid in digestion of food. These children often require up to 150% of the caloric intake of their peers. The nutritional recommendation for CF patients is high-calorie and high-protein. 2. A high-fat, high-carbohydrate diet is not recommended for adequate nutrition. 3. Exercise is effective in helping CF patients clear secretions. 4. Minimizing pulmonary complications is essential to a better outcome for CF patients. Compliance with CPT, nebulizer treatments, and medications are all components of minimizing pulmonary complications. 5. Medication compliance is a necessary part of maintaining pulmonary and gastrointestinal function. TEST-TAKING HINT: The test taker can eliminate answer 2 because patients are not placed on high-fat diets.

During hemodialysis, the nurse notes that a 10-year-old becomes confused and restless. The child complains of a headache and nausea and has generalized muscle twitching. This can be prevented by which of the following? 1. Slowing the rate of solute removal during dialysis. 2. Ensuring the patient is warm during dialysis. 3. Administering antibiotics before dialysis. 4. Obtaining an accurate weight the night before dialysis.

1. The child is experiencing signs of disequilibrium syndrome, which is caused by free water shifting from intravascular spaces and can be pre- vented by slowing the rate of dialysis.

Which manifestations should the nurse expect to find in a child in the early stages of acute hepatitis? 1. Nausea, vomiting, and generalized malaise. 2. Nausea, vomiting, and pain in the left upper quadrant. 3. Generalized malaise and yellowing of the skin and sclera. 4. Yellowing of the skin and sclera without any other generalized complaints.

1. The early stage of acute hepatitis is referred to as the anicteric phase, during which the child usually com- plains of nausea, vomiting, and generalized malaise.

Which causes the clinical manifestations of hydronephrosis? 1. A structural abnormality in the urinary system causes urine to back up and can cause pressure and cell death. 2. A structural abnormality causes urine to flow too freely through the urinary system, leading to fluid and electrolyte imbalances. 3. Decreased production of urine in one or both kidneys results in an electrolyte imbalance. 4. Urine with an abnormal electrolyte balance and concentration leads to increased blood pressure and subsequent increased glomerular filtration rate.

1. Hydronephrosis is due to a structural abnormality in the urinary system, causing urine to back up, leading to pressure and potential cell death.

Which statement by a parent is most consistent with minimal change nephrotic syndrome (MCNS)? 1. "My child missed 2 days of school last week because of a really bad cold." 2. "After camping last week, my child's legs were covered in bug bites." 3. "My child came home from school a week ago due to vomiting and stomach cramps." 4. "We have a pet turtle but no one washes their hands after playing with the turtle."

1. An upper respiratory infection often precedes MCNS by a few days.

A 13-month-old is discharged following repair of his epispadias. Which statement made by the parents indicates they understand the discharge teaching? 1. "If a mucous plug forms in the urinary drainage tube, we will irrigate it gently to prevent a blockage." 2. "If a mucous plug forms in the urinary drainage tube, we will allow it to pass on its own because this is a sign of healing." 3. "We will make sure the dressing is loosely applied to increase the toddler's comfort." 4. "If we notice any yellow drainage, we will know that everything is healing well."

1. Any mucous plugs should be removed by irrigation to prevent blockage of the urinary drainage system.

A child with hemolytic uremic syndrome (HUS) is very pale and lethargic. Stools have progressed from watery to bloody diarrhea. Blood work indicates low hemoglo- bin and hematocrit levels. The child has not had any urine output in 24 hours. The nurse expects administration of blood products and what else to be added to the plan of care? 1. Initiation of dialysis. 2. Close observation of the child's hemodynamic status. 3. Diuretic therapy to force urinary output. 4. Monitoring of urinary output.

1. Because the child is symptomatic, dialysis is the treatment of choice.

The parent of a 4-month-old with cystic fibrosis (CF) asks the nurse what time to begin the child's first chest physiotherapy (CPT) each day. Which is the nurse's best response? 1. "Thirty minutes before feeding the child breakfast." 2. "After deep-suctioning the child each morning." 3. "Thirty minutes after feeding the child breakfast." 4. "Only when the child has congestion or coughing."

1. CPT should be done in the morning prior to feeding to avoid the risk of the child vomiting. TEST-TAKING HINT: Answer 4 can be eliminated because of the word "only." There are very few times in health care when an answer will be "only." Answer 3 can be eliminated when one considers the risk of vomiting and aspiration that may occur if percussion is performed following eating.

Who is at the highest priority to receive the flu vaccine? 1. A healthy 8-month-old who attends day care. 2. A 3-year-old who is undergoing chemotherapy. 3. A 7-year-old who attends public school. 4. An 18-year-old who is living in a college dormitory

1. Children between the ages of 6 and 23 months are at the highest risk for having complications as a result of the flu. Their immune systems are not as developed, so they are at a higher risk for influenza-related hospitalizations. TEST-TAKING HINT: The test taker can eliminate answers 2 and 3 by knowing that infants and the elderly are at highest risk for complications related to the flu.

The nurse is caring for a 12-year-old receiving peritoneal dialysis. The nurse notes the return to be cloudy, and the child is complaining of abdominal pain. The child's parents ask what the next step will likely be. Which is the nurse's best response? 1. "We will probably place antibiotics in the dialysis fluid before the next dwell time." 2. "Many children experience cloudy returns. We do not usually worry about it." 3. "We will probably give your child some oral antibiotics just to make sure nothing else develops." 4. "The abdominal pain is likely due to the fluid going in too slowly. We will increase the rate of administration with the next fill."

1. Cloudy returns and abdominal pain are signs of peritonitis and are usually treated with the administration of antibiotics in the dialysis fluid.

Which is the nurse's best response to the parent of a child diagnosed with epiglottitis who asks what the treatment will be? 1. Complete a course of intravenous antibiotics. 2. Surgery to remove the tonsils. 3. 10 days of aerosolized ribavirin. 4. No intervention.

1. Epiglottitis is bacterial in nature and requires intravenous antibiotics. A 7- to 10-day course of oral antibiotics is usually ordered following the intravenous course of antibiotics. TEST-TAKING HINT: Understanding that epiglottitis is bacterial in nature will lead the test taker to choose the correct answer.

Which should be included in the plan of care for a child diagnosed with hydronephrosis? 1. Intake and output as well as vital signs should be strictly monitored. 2. Fluids and sodium in the diet should be limited. 3. Steroids should be administered as ordered. 4. Limited contact with other people to avoid infection.

1. Fluid status is monitored to ensure adequate urinary output. Assessing blood pressure monitors kidney function.

Which is a care priority for a newborn diagnosed with bladder exstrophy and a malformed pelvis? 1. Change the diaper frequently and assess for skin breakdown. 2. Keep the exposed bladder open in a warm and dry environment to avoid any heat loss. 3. Offer formula for infant growth and fluid management. 4. Cluster all care to allow the child to sleep, grow, and gain strength for the upcoming surgical repair.

1. Preventing infection from stool contamination and skin breakdown is the top priority of care.

Which would the nurse expect to hear the parents of an infant with an incarcerated hernia report? 1. Acute onset of pain, abdominal distention, and a mass that cannot be reduced. 2. Gradual onset of pain, abdominal distention, and a mass that cannot be reduced. 3. Acute onset of pain, abdominal distention, and a mass that is easily reduced. 4. Gradual onset of pain, abdominal distention, and a mass that is easily reduced.

1. Signs of an incarcerated hernia include an acute onset of pain, abdominal distention, and a mass that cannot be reduced. Other signs are bloody stools, edema of the scrotum, and a history of poor feeding.

Which statement indicates the parent needs further teaching on how to prevent his other children from contracting respiratory syncytial virus (RSV)? 1. "I should make sure that both my children receive Synagis (palivizumab) injections for the remainder of this year." 2. "I should be sure to keep my infected child away from his brother until he has recovered." 3. "I should insist that all people who come in contact with my children thoroughly wash their hands before playing with them." 4. "I should insist that anyone with a respiratory illness avoid contact with my children until well."

1. Synagis will not help the child who has already contracted the illness. Synagis is an immunization and a method of primary prevention. TEST-TAKING HINT: This question requires the test taker to understand how RSV is transmitted and how to prevent the spread of the virus.

Which causes the symptoms in testicular torsion? 1. Twisting of the spermatic cord interrupts the blood supply. 2. Swelling of the scrotal sac leads to testicular displacement. 3. Unmanaged undescended testes cause testicular displacement. 4. Microthrombi formation in the vessels of the spermatic cord causes interruption of the blood supply.

1. Testicular torsion is caused by an interruption of the blood supply due to twisting of the spermatic cord.

Which is an accurate description of a Kasai procedure? 1. A palliative procedure in which the bile duct is attached to a loop of bowel to assist with bile drainage. 2. A curative procedure in which a connection is made between the bile duct and a loop of bowel to assist with bile drainage. 3. A curative procedure in which the bile duct is banded to prevent bile leakage. 4. A palliative procedure in which the bile duct is banded to prevent bile leakage.

1. The Kasai procedure is a palliative procedure in which the bile duct is attached to a loop of bowel to assist with bile drainage.

The nurse receives a call from the mother of a 6-month-old who describes her child as alternately sleepy and fussy. She states that her infant vomited once this morning and had two episodes of diarrhea. The last episode contained mucus and a small amount of blood. She asks the nurse what she should do. Select the nurse's best response. 1. "Your infant will need to have some tests in the emergency room to determine if anything serious is going on." 2. "Try feeding your infant in about 30 minutes; in the event of repeat vomiting, bring the infant to the emergency room for some tests and intravenous rehydration." 3. "Many infants display these symptoms when they develop an allergy to the for- mula they are receiving; try switching to a soy-based formula." 4. "Do not worry about the blood and mucus in the stool; it is not unusual for in- fants to have blood in their stools because their intestines are more sensitive."

1. The infant is displaying signs of intus- susception. This is an emergency that needs to be evaluated to prevent is- chemia and perforation.

What would the nurse advise the parent of a child with a barky cough that gets worse at night? 1. Take the child outside into the more humid night air for 15 minutes. 2. Take the child to the ER immediately. 3. Give the child an over-the-counter cough suppressant. 4. Give the child warm liquids to soothe the throat.

1. The night air will help decrease subglottic edema, easing the child's respiratory effort. The coughing should diminish significantly, and the child should be able to rest comfortably. If the symptoms do not improve after taking the child outside, the parent should have the child seen by a health-care provider. TEST-TAKING HINT: The test taker must accurately identify that the question is describing a child with croup and know how croup is treated.

Which statement by the parent of a child using an albuterol inhaler leads the nurse to believe that further education is needed on how to administer the medication? 1. "I should administer two quick puffs of the albuterol inhaler using a spacer." 2. "I should always use a spacer when administering the albuterol inhaler." 3. "I should be sure that my child is in an upright position when administering the inhaler." 4. "I should always shake the inhaler before administering a dose."

1. The parent should always give one puff at a time and wait 1 minute before administering the second puff. TEST-TAKING HINT: The test taker evaluates how the parents administer the MDI.

Chronic hypertension in the child who has chronic renal failure (CRF) is due to which of the following? 1. Retention of sodium and water. 2. Obstruction of the urinary system. 3. Accumulation of waste products in the body. 4. Generalized metabolic alkalosis.

1. The retention of sodium and water leads to hypertension.

The manifestations of hemolytic uremic syndrome (HUS) are due primarily to which event? 1. The swollen lining of the small blood vessels damages the red blood cells, which are then removed by the spleen, leading to anemia. 2. There is a disturbance of the glomerular basement membrane, allowing large proteins to pass through. 3. The red blood cell changes shape, causing it to obstruct microcirculation. 4. There is a depression in the production of all formed elements of the blood.

1. The swollen lining of the small blood vessels damages the red blood cells, which are then removed by the spleen.

The nurse is caring for a child due for surgery on a Wilms tumor. The child's procedure will consist of which of the following? 1. Only the affected kidney will be removed. 2. Both the affected kidney and the other kidney will be removed in case of recurrence. 3. The mass will be removed from the affected kidney. 4. The mass will be removed from the affected kidney, and a biopsy of the tissue of the unaffected kidney will be done.

1. The treatment of a Wilms tumor involves removal of the affected kidney.

The nurse is caring for an infant diagnosed with Hirschsprung disease. The mother states she is pregnant with a boy and wants to know if her new baby will likely have the disorder. Select the nurse's best response. 1. "Genetics play a small role in Hirschsprung disease, so there is a chance the baby will develop it as well." 2. "There is no evidence to support a genetic link, so it is very unlikely the baby will also have it." 3. "It is rarely seen in boys, so it is not likely your new baby will have Hirschsprung disease." 4. "Hirschsprung disease is seen only in girls, so your new baby will not be at risk."

1. There is a genetic component to Hirschsprung disease, so any future siblings are also at risk.

Which child with asthma should the nurse see first? 1. A 12-month-old who has a mild cry, is pale in color, has diminished breath sounds, and has an oxygen saturation of 93%. 2. A 5-year-old who is speaking in complete sentences, is pink in color, is wheezing bilaterally, and has an oxygen saturation of 93%. 3. A 9-year-old who is quiet, is pale in color, and is wheezing bilaterally with an oxygen saturation of 92%. 4. A 16-year-old who is speaking in short sentences, is wheezing, is sitting upright, and has an oxygen saturation of 93%.

1. This child is exhibiting signs of severe asthma. This child should be seen first. The child no longer has wheezes and now has diminished breath signs. TEST-TAKING HINT: The test taker can eliminate answers 2, 3, and 4 by knowing that diminished breath sounds are a sign the patient has a worsening condition. The other bit of information that is essential in this problem is the child's age. The younger the child, the faster the respiratory status can diminish.

A child had a urinary tract infection (UTI) 3 months ago and was treated with an oral antibiotic. A follow-up urinalysis revealed normal results. The child has had no other problems until this visit when the child was diagnosed with another UTI. Which is the most appropriate plan? 1. Urinalysis, urine culture, and VCUG. 2. Evaluate for renal failure. 3. Admit to the pediatric unit. 4. Discharge home on an antibiotic.

1. Urinalysis and urine culture are rou- tinely used to diagnose UTIs. VCUG is used to determine the extent of urinary tract involvement when a child has a second UTI within 1 year.

The nurse is providing discharge instructions to the parents of an infant who has had surgery to open a low imperforate anus. The nurse knows that the discharge instructions have been understood when the child's parents say: 1. "We will use an oral thermometer because we cannot use a rectal one." 2. "We will call the physician if the stools change in consistency." 3. "Our infant will never be toilet-trained." 4. "We understand that it is not unusual for our infant's urine to contain stool."

2. A change in stool consistency is important to report because it could indicate stenosis of the rectum.

Which is true of a Wilms tumor? Select all that apply. 1. It is also referred to as neuroblastoma. 2. It can occur at any age but is seen most often between the ages of 2 and 5 years. 3. It can occur on its own or can be associated with many congenital anomalies. 4. It is a slow-growing tumor. 5. It is associated with a very poor prognosis.

2, 3. 1. It is referred to as a nephroblastoma, not a neuroblastoma. 2. It can occur at any age but is seen most often between the ages of 2 and 5 years. 3. It can occur on its own or can be associated with many congenital anomalies. 4. It is a tumor that grows very quickly. 5. It is associated with a very good prognosis.

A 10-kg toddler is diagnosed with acute renal failure (ARF), is afebrile, and has a 24-hour urine output of 110 mL. After calculating daily fluid maintenance, which would the nurse expect the toddler's daily allotment of fluids to be? 1. Sips of clear fluids and ice chips only. 2. 350 mL of oral and intravenous fluids. 3. 1000 mL of oral and intravenous fluids. 4. 2000 mL of oral and intravenous fluids.

2. 350 mL is approximately a third of the daily fluid requirement and is recom- mended for the child in the oliguric phase of ARF. If the child were febrile, the fluid intake would be increased.

A 3-year-old is brought to the ER with coughing and gagging. The parent reports that the child was eating carrots when she began to gag. Which diagnostic evaluation will be used to determine if the child has aspirated carrots? 1. Chest x-ray. 2. Bronchoscopy. 3. Arterial blood gas (ABG). 4. Sputum culture.

2. A bronchoscopy will allow the physician to visualize the airway and will help determine if the child aspirated the carrot. TEST-TAKING HINT: Answer 1 can be eliminated because items that are not radiopaque (opaque to x-rays) cannot be seen on an x-ray. Answers 3 and 4 can be eliminated because they do not provide confirmation regarding whether the child aspirated.

The parent of a child with cystic fibrosis (CF) is excited about the possibility of the child receiving a double lung transplant. What should the parent understand? 1. The transplant will cure the child of CF and allow the child to lead a long and healthy life. 2. The transplant will not cure the child of CF but will allow the child to have a longer life. 3. The transplant will help to reverse the multisystem damage that has been caused by CF. 4. The transplant will be the child's only chance at surviving long enough to graduate college.

2. A lung transplant does not cure CF, but it does offer the patient an opportunity to live a longer life. The concerns are that, after the lung transplant, the child is at risk for rejection of the new organ and for development of secondary infections because of the immunosuppressive therapy. TEST-TAKING HINT: Answer 4 can be eliminated because of the word "only." There are very few times in health care when an answer will be "only." Answers 1 and 3 can be eliminated if the test taker has a basic knowledge of the pathophysiology of CF.

The nurse is caring for an infant who has been diagnosed with short bowel syndrome (SBS). The parent asks how the disease will affect the child. Select the nurse's best response. 1. "Because your child has a shorter intestine than most, your child will likely experience constipation and will need to be placed on a bowel regimen." 2. "Because your child has a shorter intestine than most, he will not be able to absorb all the nutrients and vitamins in food and will need to get nutrients in other ways." 3. "Unfortunately, most children with this diagnosis do not do very well." 4. "The prognosis and course of the disease have changed because hyperalimentation is available."

2. Because the intestine is used for absorption, children with SBS usually need alternative forms of nutrition such as hyperalimentation.

The parent of a child with glomerulonephritis asks the nurse why the urine is such a funny color. Which is the nurse's best response? 1. "It is not uncommon for the urine to be discolored when children are receiving steroids and blood pressure medications." 2. "There is blood in your child's urine that causes it to be tea-colored." 3. "Your child's urine is very concentrated, so it appears to be discolored." 4. "A ketogenic diet often causes the urine to be tea-colored."

2. Blood in the child's urine causes it to be tea-colored.

Which manifestation suggests that an infant is developing necrotizing enterocolitis (NEC)? 1. Absorption of bolus orogastric feedings at a faster rate than previous feedings. 2. Bloody diarrhea. 3. Increased bowel sounds. 4. Appears hungry right before a scheduled feeding.

2. Bloody diarrhea can indicate that the infant has NEC.

The nurse is caring for a 3-year-old who had an appendectomy 2 days ago. The child has a fever of 101.8°F (38.8°C) and breath sounds are slightly diminished in the right lower lobe. Which action is most appropriate? 1. Teach the child how to use an incentive spirometer. 2. Encourage the child to blow bubbles. 3. Obtain an order for intravenous antibiotics. 4. Obtain an order for Tylenol (acetaminophen).

2. Blowing bubbles is a developmentally appropriate way to help the preschooler take deep breaths and cough.

Which statement about pneumonia is accurate? 1. Pneumonia is most frequently caused by bacterial agents. 2. Children with bacterial pneumonia are usually sicker than children with viral pneumonia. 3. Children with viral pneumonia are usually sicker than those with bacterial pneumonia. 4. Children with viral pneumonia must be treated with a complete course of antibiotics.

2. Children with bacterial pneumonia are usually sicker than children with viral pneumonia. Children with bacterial pneumonia can be treated effectively, but they require a course of antibiotics. TEST-TAKING HINT: The test taker must have an understanding of the differences between viral and bacterial infections.

A child had a tonsillectomy 6 days ago and was seen in the emergency room 4 hours ago due to post-operative hemorrhage. The parent noted that her child was "swal- lowing a lot and finally began vomiting large amounts of blood." The child's vital signs are as follows: T 99.5°F (37.5°C), HR 124, BP 84/48, and RR 26. The nurse knows that this child is at risk for which type of renal failure? 1. CRF due to advanced disease process. 2. Prerenal failure due to dehydration. 3. Primary kidney damage due to a lack of urine flowing through the system. 4. Postrenal failure due to a hypotensive state.

2. Examples of causes of prerenal failure include dehydration and hemorrhage.

Which protrusion into the groin of a female most likely causes inguinal hernias? 1. Bowel. 2. Fallopian tube. 3. Large thrombus formation. 4. Muscle tissue.

2. Fallopian tube or an ovary is the most common tissue to protrude into the groin in females.

The nurse is caring for a 7-week-old scheduled for a pyloromyotomy in 24 hours. Which would the nurse expect to find in the plan of care? 1. Keep infant NPO; begin intravenous fluids at maintenance. 2. Keep infant NPO; begin intravenous fluids at maintenance; place nasogastric tube (NGT) to low wall suction. 3. Obtain serum electrolytes; keep infant NPO; do not attempt to pass NGT due to obstruction. 4. Offer infant small frequent feedings; keep NPO 6 to 8 hours before surgery.

2. In addition to giving fluids intra- venously and keeping the infant NPO, an NGT is placed to decompress the stomach.

The clinical manifestations of minimal change nephrotic syndrome (MCNS) are due to which of the following? 1. Chemical changes in the composition of albumin. 2. Increased permeability of the glomeruli. 3. Obstruction of the capillaries of the glomeruli. 4. Loss of the kidney's ability to excrete waste and concentrate urine.

2. Increased permeability of the glomeruli in MCNS allows large substances such as protein to pass through and be excreted in the urine.

The nurse is caring for a newborn who has just been diagnosed with tracheo - esophageal fistula and is scheduled for surgery. Which should the nurse expect to do in the pre-operative period? 1. Keep the child in a monitored crib, obtain frequent vital signs, and allow the parents to visit but not hold their infant. 2. Administer intravenous fluids and antibiotics. 3. Place the infant on 100% oxygen via a non-rebreather mask. 4. Have the mother feed the infant slowly in a monitored area, stopping all feedings 4 to 6 hours before surgery.

2. Intravenous fluids are administered to prevent dehydration because the infant is NPO. Intravenous antibiotics are administered to prevent pneumonia because aspiration of secretions is likely.

A school-age child has been diagnosed with nasopharyngitis. The parent is concerned because the child has had little or no appetite for the last 24 hours. Which is the nurse's best response? 1. "Do not be concerned; it is common for children to have a decreased appetite during a respiratory illness." 2. "Be sure your child is taking an adequate amount of fluids. The appetite should return soon." 3. "Try offering the child some favorite food. Maybe that will improve the appetite." 4. "You need to force your child to eat whatever you can; adequate nutrition is essential."

2. It is common for children to have a decreased appetite when they have a respiratory illness. The nurse is appropriately instructing the parent that the child will be fine by taking in an adequate amount of fluid. TEST-TAKING HINT: Answer 4 can be eliminated because one should not force thechild to eat. If the word had been "encourage," it would have been a better choice, although still not the best answer. Answer 1 can be eliminated because the nurse did not inform the parent of the importance of maintaining adequate fluid intake.

Which position would be most comfortable for a child with left-sided pneumonia? 1. Trendelenburg. 2. Left side. 3. Right side. 4. Supine.

2. Lying on the left side may provide the patient with the most comfort. Lying on the left splints the chest and reduces the pleural rubbing. TEST-TAKING HINT: The test taker can eliminate answers 1 and 4 because neither of them would improve the child's respiratory effort. Both these positions may actually cause the patient increased respiratory distress.

Which combination of signs is commonly associated with glomerulonephritis? 1. Massive proteinuria, hematuria, decreased urinary output, and lethargy. 2. Mild proteinuria, increased urinary output, and lethargy. 3. Mild proteinuria, hematuria, decreased urinary output, and lethargy. 4. Massive proteinuria, decreased urinary output, and hypotension.

3. Mild-to-moderate proteinuria, hematuria, decreased urinary output, and lethargy are common findings in glomerulonephritis.

The nurse is caring for a 5-year-old who has just returned from having an appendectomy. Which is the optimal way to manage pain? 1. Intravenous morphine as needed. 2. Liquid Tylenol (acetaminophen) with codeine as needed. 3. Morphine administered through a PCA pump. 4. Intramuscular morphine as needed.

3. Morphine administered through a PCA pump offers the child control over managing pain. The PCA pump also has the benefit of offering a basal rate as well as an as-needed rate for optimal pain management.

Which is diagnostic for epiglottitis? 1. Blood test. 2. Throat swab. 3. Lateral neck x-ray of the soft tissue. 4. Signs and symptoms.

3. A lateral neck x-ray is a definitive test to diagnose epiglottitis. The child is at risk for complete airway obstruction and should always be accompanied by a nurse to the x-ray department. TEST-TAKING HINT: The test taker can eliminate answers 1, 2, and 4 because epiglottitis is diagnosed by lateral neck films.

Which finding requires immediate attention in a child with glomerulonephritis? 1. Sleeping most of the day and being very "cranky" when awake; blood pressure is 170/90. 2. Urine output is 190 mL in an 8-hour period and is the color of Coca-Cola. 3. Complaining of a severe headache and photophobia. 4. Refusing breakfast and lunch and stating he "just is not hungry."

3. A severe headache and photophobia can be signs of encephalopathy due to hypertension, and the child needs immediate attention.

Which would the nurse expect to be included in the diagnostic workup of a child with suspected celiac disease? 1. Obtain complete blood count and serum electrolytes. 2. Obtain complete blood count and stool sample; keep child NPO. 3. Obtain stool sample and prepare child for jejunal biopsy. 4. Obtain complete blood count and serum electrolytes; monitor child's response to gluten-containing diet.

3. A stool sample for analysis of fat and a jejunal biopsy can confirm the diagnosis.

A child is diagnosed with chronic constipation that has been unresponsive to dietary and activity changes. Which pharmacological measure is most appropriate? 1. Natural supplements and herbs. 2. Stimulant laxative. 3. Osmotic agent. 4. Pharmacological measures are not used in pediatric constipation.

3. A stool softener is the drug of choice because it will lead to easier evacuation.

The nurse is caring for a 5-month-old infant with a diagnosis of intussusception. The infant has periods of irritability during which the knees are brought to chest and the infant cries, alternating with periods of lethargy. Vital signs are stable and within age-appropriate limits. The physician elects to give an enema. The parents ask the purpose of the enema. Select the nurse's most appropriate response. 1. "The enema will confirm the diagnosis. If the test result is positive, your child will need to have surgery to correct the intussusception." 2. "The enema will confirm the diagnosis. Although very unlikely, the enema may also help fix the intussusception so that your child will not immediately need surgery." 3. "The enema will help confirm the diagnosis and has a good chance of fixing the intussusception." 4. "The enema will help confirm the diagnosis and may temporarily fix the intussus- ception. If the bowel returns to normal, there is a strong likelihood that the intussusception will recur."

3. In most cases of intussusception in young children, an enema is successful in reducing the intussusception.

The parents of a 4-year-old ask the nurse how to manage their child's constipation. Select the nurse's best response. 1. "Add 2 ounces of apple or pear juice to the child's diet." 2. "Be sure your child eats a lot of fresh fruit such as apples and bananas." 3. "Encourage your child to drink more fluids." 4. "Decrease bulky foods such as whole-grain breads and rice."

3. Increasing fluid consumption helps to decrease the hardness of the stool.

A 7-month-old has a low-grade fever, nasal congestion, and a mild cough. What should the nursing care management of this child include? 1. Maintaining strict bedrest. 2. Avoiding contact with family members. 3. Instilling saline nose drops and bulb suctioning. 4. Keeping the head of the bed flat.

3. Infants are nose breathers and often have increased difficulty when they are congested. Nasal saline drops and gentle suctioning with a bulb syringe are often recommended. TEST-TAKING HINT: The test taker can eliminate answer 4 given a basic understanding of interventions to improve respiratory function.

The nurse is caring for an 8-week-old infant being evaluated for pyloric stenosis. Which statement by the parent would be typical for a child with this diagnosis? 1. "The baby is a very fussy eater and just does not want to eat." 2. "The baby tends to have a very forceful vomiting episode about 30 minutes after most feedings." 3. "The baby is always hungry after vomiting so I refeed." 4. "The baby is happy in spite of getting really upset after spitting up."

3. Infants with pyloric stenosis are always hungry and often appear malnourished.

The nurse is caring for a 1-month-old term infant who experienced an anoxic episode at birth. The health-care team suspects that the infant is developing necrotizing en - terocolitis (NEC). Which would the nurse expect to be included in the plan of care? 1. Immediately remove the feeding nasogastric tube (NGT) from the infant. 2. Obtain vital signs every 4 hours. 3. Prepare to administer antibiotics intravenously. 4. Change feedings to half-strength, administer slowly via a feeding pump.

3. Intravenous antibiotics are adminis- tered to prevent or treat sepsis.

Which should be included in instructions to the parent of a child prescribed amoxicillin to treat an ear infection? 1. "Continue the amoxicillin until the child's symptoms subside." 2. "Administer an over-the-counter antihistamine with the antibiotic." 3. "Administer the amoxicillin until all the medication is gone." 4. "Allow your child to administer his own dose of amoxicillin."

3. It is essential that all the medication be given. TEST-TAKING HINT: Answer 1 can be eliminated because a course of antibiotics should always be completed as ordered, no matter what the age of the child. Answer 4 can be eliminated because children would not be expected to administer their own medications without supervision by an adult.

Which should the nurse instruct children to do to stop the spread of influenza in the classroom? 1. Stay home if they have a runny nose and cough. 2. Wash their hands after using the restroom. 3. Wash their hands after sneezing. 4. Have a flu shot annually.

3. It is essential that children wash their hands after any contact with nasopharyngeal secretions. TEST-TAKING HINT: Answers 1 and 4 can be eliminated because both situations are under parental control.

The nurse is caring for a 3-month-old infant who has short bowel syndrome (SBS) and has been receiving total parenteral nutrition (TPN). The parents ask if their child will ever be able to eat. Select the nurse's best response. 1. "Children with SBS are never able to eat and must receive all of their nutrition in intravenous form." 2. "You will have to start feeding your child because children cannot be on TPN longer than 6 months." 3. "We will start feeding your child soon so that the bowel continues to receive stimulation." 4. "Your child will start receiving tube feedings soon but will never be able to eat by mouth."

3. It is important to begin feedings as soon as the bowel is healed so that it receives stimulation and does not atrophy.

The nurse is caring for a newborn with hypospadias. His parents ask if circumcision is an option. Which is the nurse's best response? 1. "Circumcision is a fading practice and is now contraindicated in most children." 2. "Circumcision in children with hypospadias is recommended because it helps prevent infection." 3. "Circumcision is an option, but it cannot be done at this time." 4. "Circumcision can never be performed in a child with hypospadias."

3. It is usually recommended that circumcision be delayed in the child with hypospadias because the foreskin may be needed for repair of the defect.

The parent of a 7-year-old voices concern over the child's continued bed-wetting at night. The parent, on going to bed, has tried getting the child up at 11:30 p.m., but the child still wakes up wet. Which is the nurse's best response about what the parent should do next? 1. "There is a medication called DDAVP that decreases the volume of the urine. The physician thinks that will work for your child." 2. "When your child wakes up wet, be very firm, and indicate how displeased you are. Have your child change the sheets to see how much work is involved." 3. "Limit fluids in the evening, and start a reward system in which your child can choose a reward after a certain number of dry nights." 4. "Bed-wetting alarms are readily available, and most children do very well with them."

3. Limiting the child's fluids in the evening will help decrease the noctur- nal urge to void. Providing positive reinforcement and allowing the child to choose a reward will increase the child's sense of control.

The parent of a 5-year-old states that the child has been having diarrhea for 24 hours, vomited twice 2 hours ago, and now claims to be thirsty. The parent asks what to offer the child because the child is refusing Pedialyte. Select the nurse's most appropriate response. 1. "You can offer clear diet soda such as Sprite and ginger ale." 2. "Pedialyte is really the best thing for your child, who, if thirsty enough, will eventually drink it." 3. "Pedialyte is really the best thing for your child. Allow your child some choice in the way to take it by offering small amounts in a spoon, medicine cup, or syringe." 4. "It really does not matter what your child drinks as long as it is kept down. Try offering small amounts of fluids in medicine cups."

3. Pedialyte is the first choice, as recom- mended by the American Academy of Pediatrics. Offering the child appropri- ate choices may allow the child to feel empowered and less likely to refuse the Pedialyte. Small, frequent amounts are usually better tolerated.

The parent of a child diagnosed with acute renal failure (ARF) asks the nurse why peritoneal dialysis was selected instead of hemodialysis. Which is the nurse's best response? 1. "Hemodialysis is not used in the pediatric population." 2. "Peritoneal dialysis has no complications, so it is a treatment used without hesitation." 3. "Peritoneal dialysis removes fluid at a slower rate than hemodialysis, so many complications are avoided." 4. "Peritoneal dialysis is much more efficient than hemodialysis."

3. Peritoneal dialysis removes fluid at a slower rate that is more easily con- trolled than that of hemodialysis.

The nurse is caring for a 2-year-old child who was admitted to the pediatric unit for moderate dehydration due to vomiting and diarrhea. The child is restless, with periods of irritability. The child is afebrile with a heart rate of 148 and a blood pressure of 90/42. Baseline laboratory tests reveal the following: Na 152, Cl 119, and glucose 115. The parents state that the child has not urinated in 12 hours. After establishing a saline lock, the nurse reviews the physician's orders. Which order should the nurse question? 1. Administer a saline bolus of 10 mL/kg, which may be repeated if the child does not urinate. 2. Recheck serum electrolytes in 12 hours. 3. After the saline bolus, begin maintenance fluids of D5 1/4 NS with 10 mEq KCl/L. 4. Give clear liquid diet as tolerated.

3. Potassium is contraindicated because the child has not yet urinated. Potassium is not added to the maintenance fluid until kidney function has been verified.

An infant is not sleeping well, crying frequently, has yellow drainage from the ear, and is diagnosed with an ear infection. Which nursing objective is the priority for the family? 1. Educating the parents about signs and symptoms of an ear infection. 2. Providing emotional support for the parents. 3. Providing pain relief for the child. 4. Promoting the flow of drainage from the ear.

3. Providing pain relief for the infant is essential. With pain relief, the child will likely stop crying and rest better. TEST-TAKING HINT: The test taker needs to consider the needs of the child and the parent at this time. If the pain is controlled, the parents and child will both be in a better state. The other items are all essential in providing care for the child with otitis, but pain relief offers the best opportunity for the child and the parent to return to normal conditions

The nurse is caring for an infant with pyloric stenosis. The parent asks if any future children will likely have pyloric stenosis. Select the nurse's best response. 1. "You seem worried; would you like to discuss your concerns?" 2. "It is very rare for a family to have more than one child with pyloric stenosis." 3. "Pyloric stenosis can run in families. It is more common among males." 4. "Although there can be a genetic link, it is very unusual for girls to have pyloric stenosis."

3. Pyloric stenosis can run in families, and it is more common in males.

How will a child with respiratory distress and stridor and who is diagnosed with RSV be treated? 1. Intravenous antibiotics. 2. Intravenous steroids. 3. Nebulized racemic epinephrine. 4. Alternating doses of Tylenol and Motrin.

3. Racemic epinephrine promotes mucosal vasoconstriction. TEST-TAKING HINT: This is a knowledgelevel question that requires the test taker to know how RSV is treated.

The nurse is caring for a 4-year-old who weighs 15 kg. At the end of a 10-hour period, the nurse notes the urine output to be 150 mL. What action does the nurse take? 1. Notifies the physician because this urine output is too low. 2. Encourages the child to increase oral intake to increase urine output. 3. Records the child's urine output in the chart. 4. Administers isotonic fluid intravenously to help with rehydration.

3. Recording the child's urine output in the chart is the appropriate action because the urine output is within the expected range of 0.5-1 mL/kg/hr, or 75-150 mL for the 10-hour period.

A parent asks how to care for a child at home who has the diagnosis of viral tonsillitis. Which is the nurse's best response? 1. "You will need to give your child a prescribed antibiotic for 10 days." 2. "You will need to schedule a follow-up appointment in 2 weeks." 3. "You can give your child Tylenol every 4 to 6 hours as needed for pain." 4. "You can place warm towels around your child's neck for comfort."

3. Tylenol is recommended PRN for pain relief. TEST-TAKING HINT: The test taker can eliminate answer 1 by knowing that antibiotics are not given for viral illnesses. Answer 4 can be eliminated by knowing that swelling and inflammation increase with heat. Cold causes vasoconstriction of the vessels, aiding in decreasing the amount of inflammation.

Which is the nurse's best response to the parent of an infant diagnosed with the first otitis media who wonders about long-term effects? 1. "The child could suffer hearing loss." 2. "The child could suffer some speech delays." 3. "The child could suffer recurrent ear infections." 4. "The child could require ear tubes."

3. When children acquire an ear infection at such a young age, there is an increased risk of recurrent infections. TEST-TAKING HINT: Answers 1, 2, and 4 can be eliminated if the test taker understands that these are all long-term effects of recurrent ear infections. The question is asking about a single incident of otitis.

A 2-year-old has just been diagnosed with cystic fibrosis (CF). The parents ask the nurse what early respiratory symptoms they should expect to see in their child. Which is the nurse's best response? 1. "You can expect your child to develop a barrel-shaped chest." 2. "You can expect your child to develop a chronic productive cough." 3. "You can expect your child to develop bronchiectasis." 4. "You can expect your child to develop wheezing respirations."

4. Wheezing respirations and a dry, nonproductive cough are common early symptoms in CF. TEST-TAKING HINT: Answer 2 can be eliminated because of the word "chronic." "Chronic" implies that the disease process is advanced rather than in the initial stages. Answers 1 and 3 can be eliminated if the test taker has knowledge of signs and symptoms of advanced lung disease.

The parents of a 7-year-old tell the nurse they do not understand the difference be- tween chronic renal failure (CRF) and acute renal failure (ARF). Which is the nurse's best response? 1. "There really is not much difference because the terms are used interchangeably." 2. "Most children experience ARF. It is highly unusual for a child to experience CRF." 3. "CRF tends to occur suddenly and is irreversible." 4. "ARF is often reversible, whereas CRF results in permanent deterioration of kidney function."

4. ARF is often reversible, whereas CRF results in permanent deterioration of kidney function.

More education about necrotizing enterocolitis (NEC) is needed in a nursing in-service when one of the participants states: 1. "Encouraging the mother to pump her milk for the feedings helps prevent NEC." 2. "Some sources state that the occurrence of NEC has increased because so many preterm infants are surviving." 3. "When signs of sepsis appear, the infant will likely deteriorate quickly." 4. "NEC occurs only in preemies and low-birth-weight infants."

4. Although much more common in preterm and low-birth-weight infants, NEC is also seen in term infants as well.

Which is the best way to obtain a urine sample in an 8-month-old being evaluated for a urinary tract infection (UTI)? 1. Carefully cleanse the perineum from front to back, and apply a self-adhesive urine collection bag to the perineum. 2. Insert an indwelling Foley catheter, obtain the sample, and wait for results. 3. Place a sterile cotton ball in the diaper, and immediately obtain the sample with a syringe after the first void. 4. Using a straight catheter, obtain the sample, and immediately remove the catheter without waiting for the results of the urine sample.

4. An in-and-out catheterization is the best way to obtain a urine culture in a child who is not yet toilet-trained.

The nurse is caring for a 14-year-old with celiac disease. The nurse knows that the patient understands the diet instructions by ordering which of the following meals? 1. Eggs, bacon, rye toast, and lactose-free milk. 2. Pancakes, orange juice, and sausage links. 3. Oat cereal, breakfast pastry, and nonfat skim milk. 4. Cheese, banana slices, rice cakes, and whole milk.

4. Cheese, banana slices, rice cakes, and whole milk do not contain gluten.

One week after kidney transplant, a child complains about abdominal pain, and the parents note that the child has been very irritable. The nurse notes a 10% weight gain as well as elevated BUN and creatinine levels. Which of the following medications would the child most likely be taking? 1. Codeine tablets. 2. Furosemide. 3. MiraLAX powder. 4. Corticosteroids.

4. Corticosteroids are considered to be part of the antirejection regimen that is essential after a kidney transplant.

The parents of a 5-week-old have just been told that their child has cystic fibrosis (CF). The mother had a sister who died of CF when she was 19 years of age. The parents are sad and ask the nurse about the current projected life expectancy. What is the nurse's best response? 1. "The life expectancy for CF patients has improved significantly in recent years." 2. "Your child might not follow the same course that the mother's sister did." 3. "The physician will come to speak to you about treatment options." 4. The nurse answers their questions briefly, listens to their concerns, and is available later after they've processed the information.

4. The nurse's best intervention is to let the parents express their concerns and fears. The nurse should be available if the parents have any other concerns or questions or if they just need someone with whom to talk. TEST-TAKING HINT: When parents are given information that their child has a chronic life-threatening disease, they are not capable of processing all the information right away; they need time. The parents are often given more information than they can possibly understand and often just need someone to listen to their concerns and needs.

The parent of a 3-year-old is shocked to hear the diagnosis of Wilms tumor and says, "How could I have missed a lump this big?" Which is the nurse's best response? 1. "Do not be hard on yourself. It's easy to overlook something that has probably been growing for months when we see our children on a regular basis." 2. "I understand you must be very upset. Your child would have had a better prognosis had you caught it earlier." 3. "It really takes a trained professional to recognize something like this." 4. "Do not blame yourself. This mass grows so fast that it was probably not noticeable a few days ago."

4. The tumor is fast-growing and could very easily not have been evident a few days earlier.

What should be the nurse's first action with a child who has a high fever, dysphagia, drooling, tachycardia, and tachypnea? 1. Immediate IV placement. 2. Immediate respiratory treatment. 3. Thorough physical assessment. 4. Lateral neck radiographs..

4. This child is exhibiting signs and symptoms of epiglottitis and should be kept as comfortable as possible. The child should be allowed to remain in the parent's lap until a lateral neck film is obtained for a definitive diagnosis TEST-TAKING HINT: The test taker must accurately identify that the question is describing a child with epiglottitis and understand that agitation in this child can result in complete airway obstruction.

An expectant mother asks the nurse if her new baby will have an umbilical hernia. The nurse bases the response on the fact that it occurs: 1. More often in large infants. 2. In white infants more than in African American infants. 3. Twice as often in male infants. 4. More often in premature infants.

4. Umbilical hernias occur more often in premature infants.

Which intervention is most appropriate to teach the mother of a child diagnosed with a URI and a dry hacking cough that prevents him from sleeping? 1. Give cough suppressants at night. 2. Give an expectorant every 4 hours. 3. Give cold and flu medication every 8 hours. 4. Give 1/2 teaspoon of honey four to five times per day

4. Warm fluids, humidification, and honey are best treatments for a URI. TEST-TAKING HINT: The latest recommendations for treatment of URIs in children are to treat the symptoms because cough medications are not effective.


Ensembles d'études connexes

"Song of Myself" by Walt Whitman

View Set

NURB 3060 Exam 4: Assessing Eyes

View Set

BLAW: Quiz 6 - Property Law (Questions)

View Set

Middle Ages Test- (Brainpop & Flocabulary)

View Set

Ch. 16: Review of Neurotransmitters and the Autonomic Nervous System

View Set

DCF Certification Test - Child Abuse and Neglect, Identifying and Reporting Child Abuse and Neglect, CAAN Study Guide, Child Abuse and Neglect (CAAN)**

View Set

NUR 310 Week 2 Infection/Tissue Integrity

View Set

NUR242 PrepU Ch 12 Nursing management during pregnancy

View Set